Sie sind auf Seite 1von 88

IASbabas Daily Quiz

January 2, 2017

Q.1) Which of the following statements about Pradhan Mantri Surakshit Matritva
Abhiyan

a) It has been launched by the Ministry of Health & Family Welfare (MoHFW)
b) It aims to provide assured, comprehensive and quality antenatal care, free of cost,
universally to all pregnant women on the 9th of every month
c) It guarantees a minimum package of antenatal care services to women in their 2nd /
3rd trimesters of pregnancy at designated government health facilities
d) All of the above

Q.1) Solution (d)

The Pradhan Mantri Surakshit Matritva Abhiyan has been launched by the Ministry of Health
& Family Welfare (MoHFW), Government of India. The program aims to provide assured,
comprehensive and quality antenatal care, free of cost, universally to all pregnant women
on the 9th of every month.

Honble Prime Minister of India highlighted the aim and purpose of introduction of the
Pradhan Mantri Surakshit Matritva Abhiyan in the 31st July 2016 episode of Mann Ki Baat.

PMSMA guarantees a minimum package of antenatal care services to women in their 2nd /
3rd trimesters of pregnancy at designated government health facilities.

The programme follows a systematic approach for engagement with private sector which
includes motivating private practitioners to volunteer for the campaign; developing
strategies for generating awareness and appealing to the private sector to participate in the
Abhiyan at government health facilities.

Read More - https://pmsma.nhp.gov.in/about-scheme/#about

In News - http://indianexpress.com/article/india/year-ender-2016-list-of-health-
ministrys-various-initiatives/

Q.2) Consider the following statements about Longitudinal Ageing Study in India (LASI)

1. It will provide scientific data for framing evidence-based policy for elderly citizens
and help in expanding the scope of health and social security policy and programmes
for older population
2. LASI is jointly funded by the Union Ministry of Health and Family Welfare, the United
States National Institute on Ageing, and the United Nations Population Fund-India
IASbabas Daily Quiz
January 2, 2017

3. The International Institute for Population Sciences (IIPS), Mumbai in collaboration


with Harvard School of Public Health (HSPH) and University of Southern California
(USC), USA is undertaking the The Longitudinal Ageing Study in India under the
aegis of the Ministry of Union Health and Family Welfare

Which of the following statements is/are correct?

a) 1 and 2
b) 1, 2 and 3
c) Only 1
d) 2 and 3

Q.2) Solution (b)

The Union Ministry of Health & Family Welfare launched the Longitudinal Ageing Study in
India (LASI) to assess the needs of its elderly people and frame policies accordingly. The
survey will take into account study on 60,000 elderly people over 25 years plan and will be
the largest survey of its kind.

LASI project will be jointly funded by the Union Health Ministry, United Nations Population
Fund-India and United States National Institute on Ageing. The survey would be undertaken
by the Mumbai based International Institute for Population Sciences (IIPS), in collaboration
with Harvard School of Public Health (HSPH) and University of Southern California (USC). It
will provide Union Health Ministry sufficient data on older population in the country that in
turn would be used to develop a scientifically validated data needed for their health,
economic and social analysis of the older population. It will investigate various health
structures, and impact of social determinants on health of the elderly. It will provide
scientific data for framing evidence-based policy for elderly citizens and help in expanding
the scope of health and social security policy and programmes for older population.

In News - http://pib.nic.in/newsite/PrintRelease.aspx?relid=138270

Q.3) Global Financial Stability Report is released by

a) IMF
b) WTO
c) World Bank
d) UNDP
IASbabas Daily Quiz
January 2, 2017

Q.3) Solution (a)

A semi-annual report by the International Monetary Fund (IMF) that assesses the stability of
global financial markets and emerging market financing

The Global Financial Stability Report focuses on current conditions, especially financial and
structural imbalances that could risk an upset in global financial stability and access to
financing by emerging market countries.

It emphasizes the ramifications of financial and economic imbalances that are highlighted in
one of the IMF's other publications, the World Economic Outlook.

The Global Financial Stability Report replaced two previous reports by the IMF, the annual
International Capital Markets Report and the quarterly Emerging Market Financing Report.
The purpose of the replacement was to provide a more frequent assessment of the
worldwide financial markets and to focus on emerging market financing in a global context.

In News - http://indianexpress.com/article/business/banking-and-finance/financial-
stability-report-rbi-warns-of-further-rise-in-bad-loans-subdued-growth-4451164/

Q.4) Consider the following statements about Strategic Forces Command (SFC)

1. It forms part of India's Nuclear Command Authority (NCA)


2. It is responsible for the management and administration of the country's tactical and
strategic nuclear weapons stockpile
3. Nuclear-capable strategic ballistic missile Agni-IV which was recently tested, will be
operational under SFC

Which of the following statements is/are correct?

a) 1 and 2
b) 2 and 3
c) 1 and 3
d) All of the above

Q.4) Solution (d)

The Strategic Forces Command (SFC) sometimes called Strategic Nuclear Command, forms
part of India's Nuclear Command Authority (NCA). It is responsible for the management and
administration of the country's tactical and strategic nuclear weapons stockpile. It was
created on January 4, 2003.
IASbabas Daily Quiz
January 2, 2017

In News - http://economictimes.indiatimes.com/news/defence/india-successfully-test-
fires-agni-iv-missile/articleshow/56291127.cms

Q.5) Consider the following statements concerning Sikkim

1. Sikkim is Indias first fully organic state


2. Kanchenjunga National Park was inscribed in the UNESCO World Heritage List
3. Sikkim was awarded the cleanest state in India by the National Sample Survey Office
(NSSO)
4. Bermiok-Tokal Senior Secondary School in a remote village in south Sikkim became
the first school in the northeast to have smart classes and smart TVs

Which of the following statements is/are correct?

a) 1, 2 and 4
b) 1, 3 and 4
c) 1, 2, 3 and 4
d) 1, 2 and 3

Q.5) Solution (c)

2016 turned out to be a great year for Sikkim as it became Indias first fully organic state
besides being adjudged the cleanest state, overall best in education and tourist destination
while the famed Kanchenjunga National Park was inscribed in the UNESCO World Heritage
List. Around 75,000 hectares of agricultural land were gradually converted to certified
organic land by implementing practices and principles as per guidelines laid down in
National Programme for Organic Production.

2016 also saw the Pawan Chamling government amending the state gambling and casino
rules banning all locals from entering casinos in the state starting July.

All four districts of the state figured among top 10 ranked districts of hill states of the
country. Simultaneously, in October, Sikkim was also adjudged the cleanest tourist
destination in the country by the Ministry of Tourism.

Bermiok-Tokal Senior Secondary School in a remote village in south Sikkim became the first
school in the northeast to have smart classes and smart TVs. Sikkim was once again
conferred with the overall best state in education.

In News - http://indianexpress.com/article/india/sikkim-became-indias-first-fully-organic-
state-in-2016-4452765/
IASbabas Daily Quiz
January 3, 2017

Q.1) Internet of Birds (IoB) is the first platform to identify birds from the Indian
subcontinent through the power of Artificial Intelligence, Deep Learning and Image
Recognition. It is developed by

a) Indian Bird Conservation Network and CISCO


b) Accenture and the Bombay Natural History Society (BNHS)
c) Salim Ali Bird Sanctuary
d) Keoladeo National Park

Q.1) Solution (b) @upscgsdigest

IT consultancy firm Accenture and the Bombay Natural History Society have developed an
Internet of Things-based Birds platform that identifies bird species found in India.

Using Artificial Intelligence technology, including machine learning and computer vision, the
cloud-based service uses an Accenture-created image recognition and deep learning
platform to accurately identify bird species from digital photos that are uploaded by the
public.

The Internet of Birds platform, available to anyone, anywhere, for free, uses a unique
citizen crowd sourcing approach to engage more people in bird watching by identifying key
species of birds and inspiring an interest in nature conservation.

India is a biodiversity hotspot and is home to almost 12.5 per cent of the worlds avifauna,
consisting of 1,300 species. The Internet of Birds platform can initially identify
approximately 300 species but will eventually support all species found in India.

Birds are excellent indicators of their environment, providing ecological information based
on when and where they are located.

In News - http://www.thehindubusinessline.com/info-tech/now-flaunt-your-bird-
knowledge-with-iot-platform/article9455422.ece

Q.2) The Supreme Court judgement on Sadiq Ali vs Election Commission of India is
concerned with

a) Allotment of a symbol
b) Election speeches
c) Absentee voting
d) NOTA
IASbabas Daily Quiz
January 3, 2017

Q.2) Solution (a)

The symbol is not a property to be divided between co-owners. The allotment of a symbol
to the candidates set up by a political party is a legal right. And in case of a split, the
Commission has been authorised to determine which of the rival groups or section is the
party entitled to the symbol. The Commission, in resolving this dispute, does not decide as
to which group represents the party, but which group is that party, a three-judge Supreme
Court Bench of Justices H.R. Khanna, K.S. Hegde and A.N. Grover clarified the purpose of
Paragraph 15, while upholding the constitutionality of the test of majority in Sadiq Ali v.
Election Commission of India.

In News - http://www.thehindu.com/todays-paper/tp-national/Who-will-get-
%E2%80%98cycle%E2%80%99-It%E2%80%99s-poll-panel%E2%80%99s-
call/article16975517.ece

Q.3) Consider the following statements about Gyrotrons

1. Gyrotron is a device used in nuclear fusion


2. Gyrotrons produce very high-energy, high-frequency microwaves which are used for
generating the plasma

Which of the following statements is/are correct?

a) Only 1
b) Only 2
c) Both 1 and 2
d) Neither 1 nor 2

Q.3) Solution (c)

Gyrotron is a device used in nuclear fusion, a process that releases much more energy and is
far cleaner than the currently used fission process.

For nuclear fusion, India would need gyrotrons. Crucial devices in fusion reaction, gyrotrons
produce very high-energy, high-frequency microwaves which are used for generating the
plasma.

In News - http://pib.nic.in/newsite/PrintRelease.aspx?relid=155992
IASbabas Daily Quiz
January 3, 2017

Q.4) Consider the following statements about Samba Mahsuri Rice Variety

1. It is a high yielding fine grain rice variety released by Directorate of Rice Research
(DRR)
2. The variety was developed using Marker assisted selection and has three major
bacterial blight resistance genes Xa21, xa13 and xa5
3. It is resistant to bacterial blight disease

Which of the following statements is/are correct?

a) 1 and 2
b) 2 and 3
c) 1 and 3
d) All of the above

Q.4) Solution (d)

Samba Mahsuri Rice Variety Bacterial Blight Resistant

It is a high yielding fine grain rice variety released by Directorate of Rice Research.
The variety was developed using Marker assisted selection and has three major
bacterial blight resistance genes Xa21, xa13 and xa5.
It is being cultivated in ~ 90,000 hectares in the states of Andhra Pradesh, Telangana,
Karnataka and Tamil Nadu.

In News - http://pib.nic.in/newsite/PrintRelease.aspx?relid=155992

Q.5) Which of the following statements about Hunar Se Rozgar Tak is/are correct?

a) It is a flagship Programme of the Ministry of Tourism to impart employable skills to


youth in the age group of 18 to 28 years
b) Badhate Kadam I covers four hospitality trades namely Food Production, Food &
Beverages Services, Bakery and Patisserie and Housekeeping Utility
c) Badhate Kadam II provides training courses to bring up Event Facilitators, Tour
Assistants, Transfer Assistants and Office Assistants, Tourism Security Guards, Skin
Care & Spa Therapists
d) All of the above
IASbabas Daily Quiz
January 3, 2017

Q.5) Solution (d)

Hunar Se Rozgar Tak

A flagship Programme of the Ministry of Tourism to impart employable skills to


youth in the age group of 18 to 28 years.
The total number of persons trained under the Programme since its inception in the
year 2009 stood at about 263918.
In order to expand the HSRT outreach, it was opened up to implementation by
private organizations with domain specific credentials/competence in terms of two
verticals titled Badhate Kadam I and Badhate Kadam II.
Badhate Kadam I covers four hospitality trades namely Food Production, Food &
Beverages Services, Bakery and Patisserie and Housekeeping Utility
Badhate Kadam II provides training courses to bring up Event Facilitators, Tour
Assistants, Transfer Assistants and Office Assistants, Tourism Security Guards, Skin
Care & Spa Therapists.

In News - http://pib.nic.in/newsite/PrintRelease.aspx?relid=155985
IASbabas Daily Quiz
January 4, 2017

Q.1) Pravasi Bharatiya Divas is an annual global convention for the Indian diaspora. The
theme for 2017 is

a) Redefining Engagement with the Indian Diaspora


b) Engaging Diaspora: Connecting Across Generations
c) Apna Bharat, Apna Gaurav
d) None of the above

Q.1) Solution (a)

The 2017 (14th) Pravasi Bharatiya Divas will be held from 79 January 2017, in Bengaluru,
Karnataka. Theme: Redefining Engagement with the Indian Diaspora". The Ministry of
External Affairs, under the guidance of Prime Minister Narendra Modi, has been effortlessly
using digital modes to reach out to Indians abroad. The campaign Digital India is also playing
a major role in bringing the success to Pravasi Bhartiya Divas. The plenary session
Leveraging Social Media for Diaspora Connect to be held on January 9, 2017, at the Pravasi
Bharatiya Divas will look at social media as a medium in connecting the Indian Diaspora.

The day commemorates the return of Mahatma Gandhi from South Africa in Bombay on 9
January 1915.

In News - http://www.thehindu.com/news/national/Pravasi-Divas-this-year-to-highlight-
social-innovations/article16980424.ece

Q.2) Which of the following pairs is are correctly matched?

a) Ayush 64 - an ayurvedic formulation for treatment of Malaria


b) Ayush 82 - an ayurvedic formulation for management of Diabetes
c) Both (a) and (b)
d) Neither (a) nor (b)

Q.2) Solution (c)

National Research Development Corporation (NRDC), an Enterprise of the Department of


Scientific & Industrial Research, Ministry of Science & Technology, Govt. of India, New Delhi
and M/s Dabur India Ltd., New Delhi have entered into License Agreements for
commercialization of Ayush-64, an ayurvedic formulation for treatment of Malaria and
Ayush-82, an ayurvedic Formulation for management of Diabetes both developed by
Central Council for Research in Ayurvedic Sciences (CCRAS), New Delhi, an Autonomous
IASbabas Daily Quiz
January 4, 2017

body of the Ministry of AYUSH (Ayurveda, Yoga and naturopathy, Unani, Siddha and
Homeopathy).

The Ayurvedic Drug Ayush-64 is very effective for the treatment of Malaria which is one of
the most prevalent; destructive widely spread disease, well known to Ayurvedic Physicians
as Visama Jvara from ancient times. In view of its wide prevalence and drug resistant
malarial parasite, a poly-herbal non-toxic drug has been developed by CCRAS after carrying
out extensive pharmacological, toxicological and Clinical studies.

Ayush-82; an anti diabetic drug also developed by CCRAS is a combination of known and
tested hypoglycemic drugs.

The use of these two drugs would help millions of people suffering from Malaria and
Diabetes.

In News - http://pib.nic.in/newsite/PrintRelease.aspx?relid=156093

Q.3) Palk Strait is between

a) Asian Turkey & European Turkey


b) India and Sri Lanka
c) Taiwan & Mainland China
d) Japan & South Korea

Q.3) Solution (b)

In News - http://www.thehindu.com/news/international/India-promises-to-phase-out-
bottom-trawling-points-to-new-initiatives/article16978560.ece

Q.4) The Association of British Travel Agents (ABTA) has released its Travel Trends Report
2017. Which of the following Indian states has been ranked 8 on the '12 Destinations to
watch in 2017' list?

a) Jammu & Kashmir


b) Meghalaya
c) Kerala
d) Rajasthan
IASbabas Daily Quiz
January 4, 2017

Q.4) Solution (c)

Kerala has been pegged as a 'Destination to Watch' in 2017 by one of UK's largest and most
influential body of travel agents and tour operators.

'God's Own Country' is the only Indian entry in the list of 12 tourist hotspots brought out by
the Association of British Travel Agents (ABTA).

The list, featured in ABTA's 'Travel Trends Report 2017', has Kerala in the eighth spot -
higher than premier destinations like the USA, South Africa and Vietnam.

The rankings consist of locations that are expected to capture travelers' imaginations over
the next year.

In News - http://www.business-standard.com/article/pti-stories/ker-featured-in-abta-s-
12-destinations-to-watch-for-2017-117010200670_1.html

Q.5) Budget 2017 may hike capital gains tax on stocks. Consider the following statements
about Capital gain tax?

1. Any profit from the sale of a capital asset is deemed as capital gains
2. A capital asset is officially defined as any kind of property held by an assesse,
excluding goods held as stock-in-trade, agricultural land and personal effects
3. Shares and equity mutual funds alone enjoy a special dispensation on capital gains
tax

Which of the following statements is/are correct?

a) 1 and 2
b) 2 and 3
c) 1 and 3
d) All of the above

Q.5) Solution (d) @upscgsdigest


@pdf4exams
What is Capital Gains Tax?

Any profit from the sale of a capital asset is deemed as capital gains.
A capital asset is officially defined as any kind of property held by an assesse,
excluding goods held as stock-in-trade, agricultural land and personal effects.
Normally if an asset is held for less than 36 months, any gain arising from selling it is
treated as a short-term capital gain (STCG) and taxed in your hands.
IASbabas Daily Quiz
January 4, 2017

This becomes a long-term capital gain (LTCG) if the asset is held for 36 months or
more.
Shares and equity mutual funds alone enjoy a special dispensation on capital gains
tax.
In their case, a holding period of 12 months or more qualifies as long-term.
Current tax laws state LTCG arising on the sale of listed equity shares or equity
oriented mutual funds are exempt from tax if you have paid Securities Transaction
Tax (STT) on the sale transaction.
STCG from such shares and funds is also taxable at a flat 15 per cent (plus surcharge
and cess).
The short-term capital loss from financial assets can be set off against any other
capital gain.

In News - http://economictimes.indiatimes.com/news/politics-and-nation/pm-narendra-
modi-hints-at-long-term-capital-gains-tax-in-budget/articleshow/56160784.cms
IASbabas Daily Quiz
January 5, 2017

Q.1) Consider the following statements about Socio Economic and Caste Census

1. The SECC 2011 ranks households based on their socio-economic status to enable
state governments to prepare a list of families living below the poverty line
2. It also makes available information regarding the socio-economic condition and
education status of various castes and sections of the population
3. The Ministry of Rural Development will start using SECC 2011 data this year for its
National Social Assistance Programme to pay pension to rural poor and National
Rural Livelihood Mission

Which of the following statements is/are correct?

a) 1 and 2
b) 2 and 3
c) 1 and 3
d) All of the above

Q.1) Solution (d)

Government will adopt the Socio-Economic and Caste Census (SECC) instead of the poverty
line-based method to identify recipients for its pro-poor schemes. The SECC 2011 ranks
households based on their socio-economic status to enable state governments to prepare a
list of families living below the poverty line. It also makes available information regarding
the socio-economic condition and education status of various castes and sections of the
population.

The Ministry of Rural Development will start using SECC 2011 data this year for its National
Social Assistance Programme to pay pension to rural poor and National Rural Livelihood
Mission. Both schemes use BPL data to estimate the number of the poor. BPL data tells us
how many are poor and SECC who are those poor it is a more targeted and scientific
approach in ensuring the right person gets the benefit.

The Sumit Bose Committee formed to study the validity and efficiency of the SECC 2011
data in identifying the poor recently submitted its report, which is in favour of using the
information for rural development schemes. The committee also developed a formula based
on deprivation parameters to identify beneficiaries for specific schemes.

The greater the deprivation score, the higher will be the ranking of a household for getting
government assistance. The government is also identifying workers under the national rural
employment guarantee scheme who have reported deprivation in the SECC study.
IASbabas Daily Quiz
January 5, 2017

The SECC data provides age-wise classification matched with various deprivation
parameters, which will now be used for the pension programme for the rural population.
The government is likely to issue a directive to states to use the SECC data instead of the
poverty line-based measure to target beneficiaries for rural schemes.

In News - http://economictimes.indiatimes.com/news/economy/policy/socio-economic-
and-caste-census-data-to-drive-welfare-schemes/articleshow/56322159.cms

Q.2) China sent its first freight train to London in what is one of the world's longest train
rides. The train will pass through which of the following counties before it gets to London?

1. Belarus
2. Poland
3. Belgium
4. Kyrgyzstan
5. Russia

Select the correct code

a) 1, 2, 3 and 4
b) 1, 2, 3 and 5
c) 1, 2 and 5
d) 2, 3 and 5

@upscgsdigest
Q.2) Solution (b) @pdf4exams
The train left Yiwu West Railway Station in Zhejiang province Sunday and is headed for the
British capital.

The train is hauling household goods, bags, suitcases and garments, among other items, and
will pass through Kazakhstan, Russia, Belarus, Poland, Germany, Belgium and France before
it gets to London.

The train is part of Chinese President Xi Jinping's vision for "One Belt, One Road" -- dubbed
by some as the new silk road. It's China's infrastructure initiative, which Xi hopes will
improve China's economic ties with Europe, Asia and the Middle East.
IASbabas Daily Quiz
January 5, 2017

In News - http://indianexpress.com/article/india/chinas-freight-train-to-london-another-
step-in-exploring-ancient-trade-routes-4456591/

Q.3) The theme for 104th Indian Science Congress is

a) Science and Technology for National Development


b) Science and Technology for Indigenous Development in India
c) Science and Technology for Human Development
d) Innovations in Science & Technology for Inclusive Development

Q.3) Solution (a)

The theme for the five-day event is Science and Technology for National Development.

The 104th Indian Science Congress will also witness Children Science Congress to be held
from January 4 to 6 at Sri Padmavati Mahila Viswa Vidyalayam aimed at highlighting the
need to create awareness about science among children and elders.

In News - http://www.financialexpress.com/india-news/prime-minister-narendra-modi-
to-inaugurate-104th-indian-science-congress-in-andhra-pradesh/494892/
IASbabas Daily Quiz
January 5, 2017

Q.4) Between 2008 and 2015, India purchased defence equipments worth USD 34 billion,
which is a distant second after Saudi Arabias USD 93.5 billion, said the report
Conventional Arms Transfers to Developing Nations 2008-2015. The report is released by

a) Institute of Economic and Peace


b) NATO
c) Congressional Research Service
d) Peninsula Shield Force

Q.4) Solution (c)

The report titled Conventional Arms Transfers to Developing Nations, 20082015 and
prepared by the nonpartisan Congressional Research Service, a division of the Library of
Congress.

In News - http://www.thehindu.com/news/international/US-tops-weapons-sales-
globally-for-2015-Congressional-report/article16949771.ece

Q.5) Consider the following statements about Centre for Cellular and Molecular Biology
(CCMB)

1. It is an Indian Biotechnology research establishment of the Council of Scientific and


Industrial Research located in Hyderabad
2. The National Biosafety level - 4 Containment Facility for Human Infectious Diseases,
is located in CCMB
3. iHUB, the innovation centre to promote start-ups in the biology space is created by
CCMB

Which of the following statements is/are correct?

a) 1 and 2
b) 2 and 3
c) 1 and 3
d) All of the above

Q.5) Solution (d)

It is an Indian Biotechnology research establishment of the Council of Scientific and


Industrial Research located in Hyderabad and a designated Centre of Excellence for Global
IASbabas Daily Quiz
January 5, 2017

Molecular and Cell Biology Network, UNESCO. The National Biosafety level - 4 Containment
Facility for Human Infectious Diseases, is located in CCMB. iHUB, the innovation centre to
promote start-ups in the biology space is created by CCMB.

In News - http://www.thehindubusinessline.com/info-tech/4-startups-ink-agreements-
with-ccmbs-ihub/article9455043.ece
IASbabas Daily Quiz
January 6, 2017

Q.1) Consider the following statements about Indian Skill Development Services (ISDS)

1. ISDS has been created for Training Directorate of the Ministry of Skill Development
and Entrepreneurship.
2. ISDS will be a Group A service in which the induction will take place through Indian
Civil Service Examination conducted by UPSC

Which of the following statements is/are correct?

a) Only 1
b) Only 2
c) Both 1 and 2
d) Neither 1 nor 2

Q.1) Solution (a)

ISDS has been created for Training Directorate of the Ministry of Skill Development and
Entrepreneurship.

ISDS will be a Group A service in which the induction will take place through Indian
Engineering Service Examination conducted by UPSC

In News - http://www.financialexpress.com/economy/government-issues-notification-of-
indian-skill-development-service-here-is-all-you-need-to-know/496745/

Q.2) Consider the following statements about Income Tax Settlement Commission.

1. It is a premier Alternative Dispute Resolution (ADR) body in India


2. Its mandate is to resolve tax disputes in respect of Indian Income Tax & Wealth Tax
Laws between the two disputing parties
3. It was set up in on the recommendations of the Direct Taxes Enquiry Committee
(1971) set up under the Chairmanship of Justice K.N. Wanchoo

Which of the following statements is/are correct?

a) 1 and 2
b) 2 and 3
c) 1 and 3
d) All of the above
IASbabas Daily Quiz
January 6, 2017

Q.2) Solution (d)

Income Tax Settlement Commission is a premier Alternative Dispute Resolution (ADR) body
in India. Its mandate is to resolve tax disputes in respect of Indian Income Tax & Wealth Tax
Laws between the two disputing parties, Income Tax Department on one side and litigating
tax payer on the other.

This institution was set up in 1976 by the Central Government on the recommendations of
the Direct Taxes Enquiry Committee (1971) set up under the Chairmanship of Justice K.N.
Wanchoo, the retired Chief Justice of the Supreme Court of India. The Wanchoo Committee
had conceived of the Settlement Commission as a mechanism to allow a one-time tax
evader or an unintending defaulter to make clean breast of his affairs.

The settlement mechanism allows taxpayers to disclose additional Income before it over
and above what has been already disclosed before the Income tax Department. The
applicant has to pay full amount of tax and interest on the additional income disclosed
before the Commission, before filing the application. The Commission then decides upon
the admissibility of the application and in case of admitted applications, carries out the
process of settlement in a time bound manner by giving opportunity to both parties.

The Commission is required to pass the Settlement order within 18 months of filling of the
application. It has wide power of granting immunity from Penalty and prosecution, which
are major sources of litigation. The orders passed by the Commission are final and
conclusive.

In News - http://indianexpress.com/article/india/sahara-gets-immunity-tax-panel-
accepts-its-claim-that-seized-papers-not-evidence-kejriwal-rahul-gandhi-raid-4459535/

Q.3) Japan has decided to be associated with development of which of the following cities
as smart cities India?

1. Aurangabad
2. Ahmedabad
3. Varanasi
4. Noida

Select the correct code:

a) 1, 3 and 4
b) 2 and 3
c) 1 and 3
d) 1, 2 and 3
IASbabas Daily Quiz
January 6, 2017

Q.3) Solution (b)

Japan has decided to be associated with development of cities like Chennai, Ahmedabad
and Varanasi as smart cities in the country.

In News - http://www.financialexpress.com/india-news/japan-comes-forward-to-partner-
india-in-building-smart-cities/497942/

Q.4) Consider the following statements about Council for Trade Development and
Promotion

1. It was constituted in order to ensure a continuous dialogue with State Governments


and UTs on measures for providing an International trade enabling environment in
the States and to create a framework for making the States active partners in
boosting Indias exports
2. Union Minister of Commerce and Industry is the chairperson on the council

Which of the following statements is/are correct?

a) Only 1
b) Only 2
c) Both 1 and 2
d) Neither 1 nor 2

Q.4) Solution (c)

It has been decided to constitute the Council for Trade Development and Promotion with
immediate effect in order to ensure a continuous dialogue with State Governments and UTs
on measures for providing an International trade enabling environment in the States and to
create a framework for making the States active partners in boosting Indias exports.2. The
compositions of the Council for Trade Development and Promotion would be as below -
IASbabas Daily Quiz
January 6, 2017

3. The Terms of Reference for the Council for Trade Development and Promotion would be
as follows:

i. To provide a platform to State Governments and UTs for articulating their


perspectives on trade policy;
ii. To provide a platform to Government of India for apprising State Governments and
UTs about international developments affecting Indias trade potential and
opportunities and to prepare them to deal with evolving situation;
iii. To help State Governments develop and pursue export strategies in line with
national Foreign Trace Policy;
iv. To provide a platform for deliberation on the need for infrastructure relevant for
promoting trade and for identification of impediments and infrastructure gaps which
adversely affect Indias exports;
v. To facilitate a mechanism for discussion on operationalization of trade
infrastructure.

The Council will be recommendatory in nature and will meet at least once every year. The
meetings of the Council will be held at Delhi or anywhere in the country. Records of the
discussion will be kept.5. The Council will be served by the Joint Secretary, in-charge of
mainstreaming work.

In News - http://pib.nic.in/newsite/PrintRelease.aspx?relid=156156

Q.5) Which of the following armies is first foreign force to take part in an Republic Day
parade which showcases the military might and cultural diversity of the Indian republic?

a) French Army
b) UAE Army
c) Japanese Army
IASbabas Daily Quiz
January 6, 2017

d) German Army

@upscgsdigest
Q.5) Solution (a) @pdf4exams

The French Army created history last year by being the first foreign force to take part in an
RD parade which showcases the military might and cultural diversity of the Indian republic.
French President Francois Hollande was the chief guest last year.

In News - http://www.thehindu.com/news/national/article16992800.ece
IASbabas Daily Quiz
January 9, 2017

Q.1) Mission Lucy and Mission Psyche is concerned with

a) Asteroids
b) Mars
c) Moon
d) None of the above

Q.1) Solution (a)

The Lucy mission named for a famous, critical hominin fossil set found in Ethiopia in 1974
will involve sending a robotic spacecraft to study Jupiters so-called Trojan asteroids.
These are thought to be relics of a much earlier era in the history of the solar system.

Psyche mission aims to explore a huge, one-of-a-kind metal asteroid, called 16 Psyche, that
is about three times farther away from the sun than the Earth is.

Most asteroids are rocky or icy but this one is thought to be mostly of iron and nickel, like
the Earths core.

In News - http://www.livemint.com/Science/qe7dSeoLqP0HlM1eVJyDWO/Nasa-
announces-2-missions-to-study-early-solar-system.html

Q.2) Consider the following statements about Telecom Disputes Settlement and Appellate
Tribunal (TDSAT)

1. It has been set up under Section 14 of the Telecom Regulatory Authority of India Act,
1997 by TRAI (Amendment) Act, 2000
2. It was established to adjudicate disputes and dispose of appeals with a view to
protect the interests of service providers and consumers of the telecom sector and
to promote and ensure orderly growth of the telecom sector.

Which of the following statements is/are correct?

a) Only 1
b) Only 2
c) Both 1 and 2
d) Neither 1 nor 2
IASbabas Daily Quiz
January 9, 2017

Q.2) Solution (c)

By the Amendment Act an Appellate Tribunal known as the Telecom Disputes Settlement &
Appellate Tribunal has been set up under Section 14 of the Telecom Regulatory Authority
of India Act, 1997 by TRAI (Amendment) Act, 2000) to adjudicate disputes and dispose of
appeals with a view to protect the interests of service providers and consumers of the
telecom sector and to promote and ensure orderly growth of the telecom sector.

The functions of the appellate tribunal are to adjudicate any dispute between a licensor and
licensee, between two or more service providers, between a service provider and a group of
consumers, and to hear and dispose of appeals against any decision or order of TRAI, the
appellate tribunal consists of Chairperson and two Members.

In News - http://economictimes.indiatimes.com/news/economy/policy/decide-on-jio-
case-within-a-reasonable-time-tdsat-to-trai/articleshow/56380293.cms

Q.3) Consider the following statements about Larsen Ice Shelf

1. It is a series of three shelves


2. It is a long, fringing ice shelf in the Arctic Ocean
3. Larsen A is the largest which is poised to break off

Which of the following statements is/are correct??

a) 1 and 2
b) Only 1
c) 1 and 3
d) 2 and 3

Q.3) Solution (b)

The Larsen Ice Shelf is a long, fringing ice shelf in the northwest part of the Weddell Sea,
extending along the east coast of the Antarctic Peninsula from Cape Longing to the area just
southward of Hearst Island. It is named for Captain Carl Anton Larsen, the master of the
Norwegian whaling vessel Jason, who sailed along the ice front as far as 6810' South during
December 1893. In finer detail, the Larsen Ice Shelf is a series of three shelves that occupy
(or occupied) distinct embayments along the coast. From north to south, the three
segments are called Larsen A (the smallest), Larsen B, and Larsen C (the largest) by
researchers who work in the area.
IASbabas Daily Quiz
January 9, 2017

In News - http://economictimes.indiatimes.com/news/environment/global-
warming/huge-antarctic-ice-block-set-to-break-off-scientists/articleshow/56392080.cms

Q.4) Which of the following committee has been constituted to draft National Sports
Development Code

a) Vijay Kelkar Committee


b) Shanta Kumar Committee
c) Nanavati Commission
d) Injeti Srinivas Committee

Q.4) Solution (d)

The committee will be headed by Sports Secretary Injeti Srinivas

To make recommendations for bringing out a comprehensive National Sports Development


Code across all disciplines

It will study issues related to sports governance, including court judgments and best
international practices

Former Olympic champion Abhinav Bindra, legendary badminton player Prakash Padukone
and athlete Anju Bobby George are a part of the nine-member government panel

International hockey federation (FIH) president Narinder Batra, gymnast Dipa Karmakars
coach Bishweshwar Nandi, lawyer Nandan Kamath and The Hindu sports journalist Vijay
Lokapally are also a part of the committee. The Joint Secretary Sports (MYAS) is the ninth
member

In News - http://www.thehindu.com/news/national/Govt.-constitutes-panel-to-draft-
National-Sports-Development-Code/article16993002.ece

Q.5) Consider the following statements about Central Statistics Office (CSO)

1. The National Accounts Division (NAD) of CSO is responsible for the preparation of
national accounts, which includes Gross Domestic Product, Government and Private
Final Consumption Expenditure, Fixed Capital Formation and other macro-economic
aggregates.
2. The Economic Statistics Division (ESD) of CSO conducts Economic Censuses and
Annual Surveys of Industries (ASI), compiles All India Index of Industrial
IASbabas Daily Quiz
January 9, 2017

Production(IIP), Energy Statistics and Infrastructure Statistics, and develops


classifications like, National Industrial Classification (NIC) and National Product
Classification (NPC)
3. Coordination and Publications Division (CAP) of CSO is a nodal Division for
administering the Collection of Statistics Act, 2008 and coordination of follow-up on
the implementation of recommendations of NSC recommendations

Which of the following statements is/are correct?

a) 1 and 2
b) 2 and 3
c) 1 and 3
d) All of the above

Q.5) Solution (d)

The Central Statistics Office coordinates the statistical activities in the country and evolves
statistical standards. It is headed by a Director General assisted by 5 Additional Director
Generals. CSO has the following Divisions:

National Accounts Division (NAD): This Division is responsible for the preparation of
national accounts, which includes Gross Domestic Product, Government and Private Final
Consumption Expenditure, Fixed Capital Formation and other macro-economic aggregates.
The Division brings out an annual publication, titled National Accounts Statistics,
containing these statistics. Other important activities of the Division are: (i) preparation of
quarterly estimates of Gross Domestic Product (GDP) at current and constant prices, (ii)
estimation of Capital Stock and Consumption of Fixed Capital, (iii) estimation of State-wise
Gross Value Added and Gross Fixed Capital Formation of supra-regional sectors of Railways,
Communication, Banking & Insurance and Central Government Administration (iv) Input-
Output Transaction Tables (IOTT) and (v) preparation of comparable estimates of State
Domestic Product (SDP).

Social Statistics Division (SSD): This Division is entrusted with Statistical monitoring of the
Millennium Development Goals, Environmental Economic Accounting, Grant-in-aid for
research, workshop/seminars/conferences in Official/Applied Statistics,
National/International awards for Statisticians, National Data Bank (NDB) on socio-religious
categories, Basic Statistics for Local Level Development (BSLLD) Pilot scheme, Time-use
survey and release of regular and ad-hoc statistical publications.

Economic Statistics Division (ESD): This Division conducts Economic Censuses and Annual
Surveys of Industries (ASI), compiles All India Index of Industrial Production (IIP), Energy
IASbabas Daily Quiz
January 9, 2017

Statistics and Infrastructure Statistics, and develops classifications like, National Industrial
Classification (NIC) and National Product Classification (NPC)

Training Division: This Division is primarily responsible for the training manpower in
theoretical and applied statistics to tackle the emerging challenges of data collection,
collation, analysis and dissemination required for evidence based policy making as also for
planning, monitoring and evaluation. The Division also looks after the National Statistical
Systems Training Academy (NSSTA), which is a premier Institute fostering human resource
development in official statistics in India as well as at international level, particularly
amongst developing and SAARC countries.

Coordination and Publications Division (CAP): The Division looks after co-ordination work
within CSO as well as with the line Ministries and State/UT Governments in statistical
matters, organizes Conference of Central and State Statistical Organizations (COCSSO) and
Statistics Day every year, prepares Results Framework Document (RFD), Citizens/Clients
Charter and Annual Action Plan, Outcome Budget and Annual Plan of the Ministry. The
Division is also responsible for implementation of Capacity Development Scheme and
Support for Statistical Strengthening (SSS) , a Central Sector Scheme aimed at improving the
Statistical Capacity and Infrastructure of the State Statistical System for Collecting,
Compiling and Disseminating relevant and reliable official statistics for policy making and to
promote their usage at the State/District and Block Levels. It is a nodal Division for
administering the Collection of Statistics Act, 2008 and coordination of follow-up on the
implementation of recommendations of NSC recommendations. The administrative work
relating to Indian Statistical Institute (ISI) is also looked after by CAP Division.

In News - http://www.hindustantimes.com/business-news/gdp-estimates-out-today-
why-note-ban-might-not-have-an-impact/story-8IVLvB5Tu6rTz6Ln7O5LAO.html
IASbabas Daily Quiz
January 10, 2017

Q.1) Consider the following statements about Banks Board Bureau (BBB)

1. It decides on management and board-level appointments in public sector banks


2. It was originally proposed by the PJ Nayak Committee
3. It is based in Mumbai

Which of the following statements is/are correct?

a) 1 and 2
b) 2 and 3
c) 1 and 3
d) All of the above

Q.1) Solution (d)

It decides on management and board-level appointments in public sector banks, is based in


Mumbai.

The BBB, originally proposed by the PJ Nayak Committee, was proposed to review
governance issues in the banking sector. It is headed by former Comptroller and Auditor
General Vinod Rai.

In News - http://www.thehindu.com/todays-paper/tp-business/New-norms-likely-for-
top-PSU-bank-posts/article16995876.ece

Q.2) Consider the following statements about Presumptive Taxation Scheme (PTS)

1. Under PTS, eligible professionals and businesses can compute income on an


estimated basis under section 44ADA and 44AD of the Income Tax Act, 1961,
respectively, at a minimum prescribed rate
2. The threshold under PTS for eligible businesses is Rs.1 crore
3. Limited liability partnership (LLP) firms are not allowed to adopt this scheme

Which of the following statements is/are correct?

a) 1 and 2
b) 1 and 3
c) 2 and 3
d) All of the above
IASbabas Daily Quiz
January 10, 2017

Q.2) Solution (b)

Under PTS, eligible professionals and businesses can compute income on an estimated basis
under section 44ADA and 44AD of the Income Tax Act, 1961, respectively, at a minimum
prescribed rate.

The threshold under PTS for eligible businesses has been raised from Rs.1 crore to Rs.2
crore. Professionals with less than Rs.50 lakh of gross receipts in a financial year are also
now under the umbrella of PTS.

The scheme can be adopted by an eligible resident individual, a resident Hindu undivided
family (HUF) and resident partnership firm. However, limited liability partnership (LLP) firms
are not allowed to adopt this scheme.

Read More - http://www.livemint.com/Money/fsqjFKoQCT7zHVKtN2MeqK/How-the-


presumptive-tax-scheme-helps-you.html

In News - http://www.livemint.com/Opinion/ElARwRICgIsmgJsWae8xLK/How-to-go-
about-chasing-black-money.html

Q.3) Recently, Supreme Court has reiterated the necessity to implement the Vote Verifier
Paper Audit Trail (VVPAT) in electronic voting machines (EVMs). Consider the following
statements about Vote Verifier Paper Audit Trail (VVPAT)

1. In the VVPAT system, when a voter presses the button for a candidate of his choice
in the EVM, a paper ballot containing the serial number, name of the candidate and
poll symbol would be printed for the voter
2. Supreme Court had directed the Election Commission to introduce the paper trail in
EVMs in a phased manner for the general elections in 2014

Which of the following statements is/are correct?

a) Only 1
b) Only 2
c) Both 1 and 2
d) Neither 1 nor 2
IASbabas Daily Quiz
January 10, 2017

Q.3) Solution (c)

The Supreme Court has reiterated the necessity to implement the Vote Verifier Paper Audit
Trail (VVPAT) in electronic voting machines (EVMs) to ensure 100 per cent transparency in
elections.

In the VVPAT system, when a voter presses the button for a candidate of his choice in the
EVM, a paper ballot containing the serial number, name of the candidate and poll symbol
would be printed for the voter.

The VVPAT is intended as an independent verification system for voting machines designed
to allow voters to verify that their votes were cast correctly, to detect possible election
fraud or malfunction and to provide a means to audit the stored electronic results.

This is the second time the Supreme Court is insisting on the implementation of VVPAT in
EVMs.

In 2013, the apex court, on a petition filed by BJP leader Subramanian Swamy, had directed
the Election Commission to introduce the paper trail in EVMs in a phased manner for the
general elections in 2014

In its recent order, Justice Gogoi directed the EC to bring on record the approximate time
within which the entire system of the VVPAT can be introduced, subject to grant of sanction
and release of funds as and when required.

Read More - http://www.elections.in/political-corner/what-is-vvpat-voter-verified-paper-


audit-trail/

In News - http://www.thehindu.com/news/national/SC-wants-paper-trail-in-EVMs-for-
fair-poll/article17009568.ece

Q.4) Which of the following statements about RE based Micro and Mini Grids is/are
correct?

a) A Mini Grid is defined as a system having a RE based electricity generator with


capacity of 10KW and above
b) A Micro Grid system is similar to a mini grid but having a RE based generation
capacity of below 10KW
c) The Public Distribution Network of a mini grid can be designed to carry either
Alternating Current or Direct Current (AC or DC)
d) All of the above
IASbabas Daily Quiz
January 10, 2017

Q.4) Solution (d)

A Mini Grid is defined as a system having a RE based electricity generator (with capacity of
10KW and above), and supplying electricity to a target set of consumers (residents for
household usage, commercial, productive, industrial and institutional setups etc.) through a
Public Distribution Network (PDN). A Micro Grid system is similar to a mini grid but having
a RE based generation capacity of below 10KW. Micro and mini grids2 generally operate in
isolation to the electricity networks of the DISCOM grid (standalone), but can also
interconnect with the grid to exchange power. If connected to grid they are termed as grid
connected mini/ micro grid.

A mini grid is a larger system that converts direct current (DC) to alternating current (AC)
and it provides safety as per REC and CEA standards.

Draft National Policy on RE based Mini/Micro grids - http://mnre.gov.in/file-


manager/UserFiles/draft-national-Mini_Micro-Grid-Policy.pdf

In News - http://www.thehindu.com/business/Economy/Minigrids-can-power-rural-
economic-activity-says-Smart-Power%E2%80%99s-Mukherjee/article17009311.ece

Q.5) ICT Development Index (IDI) is released by

a) International Telecommunications Unit (ITU)


b) Amazon Web Services
c) Global Alliance for ICT and Development
d) International Institute for Communication and Development (IICD)

Q.5) Solution (a)

The ICT Development Index (IDI) rankings compare the state of ICT development across
countries. It is published annually by the UN ITU. The IDI is based on 11 ICT indicators,
grouped in three sub-indices: access, use and skills. It is standard tool that governments,
operators, researchers, development agencies and others can use to measure the digital
divide and compare ICT performance within and across countries.

Based on Indias IDI data, it is ranked 138 worldwide, behind nations such as Gabon, Nigeria
and Zimbabwe, which rank significantly lower on economic and human development.

In News - http://www.livemint.com/Opinion/Mj3KgqCK1cZ2hbZYwbvE9H/Will-Aadhaar-
help-the-poor-become-cashless.html
IASbabas Daily Quiz
January 11, 2017

Q.1) Consider the following statements about Debts Recovery Tribunals and Appellate
Tribunals.

1. It provides expeditious adjudication and recovery of debts due to Banks and


Financial Institution
2. The Debts Recovery Tribunals (DRTs) and Debts Recovery Appellate Tribunal (DRATs)
were established under the Recovery of Debts Due to Banks and Financial
Institutions Act (RDDBFI Act), 1993.
3. Presently 34 DRTs and 5 DRATs are functional

Which of the following statements is/are correct?

a) 1 and 2
b) 2 and 3
c) 1 and 3
d) All of the above

Q.1) Solution (d)

The Debts Recovery Tribunals (DRTs) and Debts Recovery Appellate Tribunal (DRATs) were
established under the Recovery of Debts Due to Banks and Financial Institutions Act (RDDBFI
Act), 1993 with the specific objective of providing expeditious adjudication and recovery of
debts due to Banks and Financial Institution. Presently 34 DRTs and 5 DRATs are
functioning in India.6 new DRTs are also being established at Bengaluru, Chandigarh, Dehra
Dun, Ernakulam, Hyderabad and Siliguri.

In News - http://www.thehindu.com/news/national/SC-asks-list-of-firms-with-debts-
over-%E2%82%B9500-cr./article16984068.ece

Q.2) The Teesta River and Farkka Barrage issue is concerned with which of the following
countries?

a) Pakistan
b) Bangladesh
c) Nepal
d) China
IASbabas Daily Quiz
January 11, 2017

Q.2) Solution (b)

The Farakka Barrage was created by India in 1975 to divert water from the Ganges River
system.

Bangladesh is a lower riparian country that heavily relies on the flow of the Ganges to meet
its food and water demands; any change in the flow of the Ganges significantly affects it.

The Farakka Barrage was created by India in 1975 to divert water from the Ganges River to
the Bhagirathi-Hoogly river system. The barrage diverts water from one of the most
populated basins in the world, the Ganges-Brahmaputra-Meghna basin. There are 54
transboundary rivers between India and Bangladesh. Given the rivers long route
throughout several countries, any source of tension between India and Bangladesh
threatens food and water security for millions of people who rely on the Ganges and its
tributaries.

The Teesta which has its source in Sikkim flows through the northern part of West
Bengal in India before entering Bangladesh, where after coursing through about 45km of
irrigable land, merges with the Brahmaputra River (or Jamuna when it enters Bangladesh).
In 1983, an ad-hoc water sharing agreement was reached between India and Bangladesh,
whereby both countries were allocated 39% and 36% of the water flow respectively. The
new bilateral treaty expands upon this agreement by proposing an equal allocation of the
Teesta River.

In News - http://indianexpress.com/article/opinion/columns/india-bangladesh-relations-
treaty-terrorism-despite-history-geography-4459231/

Q.3) Which of the following is/are correctly matched?

1. Banasura laughingthrush : Wayanand


2. Travancore laughingthrush : Thiruvananthapuram
3. Nilgiri laughingthrush : Silent Valley National Park
4. Palani laughingthrush : Munnar hills

Select the correct code:

a) 1, 2 and 3
b) 2, 3 and 4
c) 1, 2, 3 and 4
d) 1 and 4
IASbabas Daily Quiz
January 11, 2017

Q.3) Solution (c)

The newly accepted species are: Banasura laughingthrush (Trochalopteron jerdoni),


which has a very restricted distribution in Wayanad district and Travancore
laughingthrush (Trochalopteron merdionale) found in Thiruvananthapuram district
Banasura species was assessed as endangered
Travancore variety was considered vulnerable
The two original species of the family were Nilgiri laughingthrush and Palani
laughingthrush
The Nilgiri species, assessed as an endangered one, is found in Silent Valley National
Park and Siruvani hills of Kerala
The near-threatened Palani laughingthrush is found mainly in Munnar hills and the
mountains of Periyar Tiger Reserve apart from Grass Hills and Palani hills in Tamil
Nadu

In News - http://www.thehindu.com/sci-tech/science/Kerala%E2%80%99s-avian-
diversity-gets-richer/article17004904.ece

Q.4) Recently scientists of the Botanical Survey of India (BSI) have found a new species of
Zingiber (commonly referred as Ginger) from

a) Andaman and Nicobar Islands


b) Lakshadweep Islands
c) Faroe Islands
d) None of the above

Q.4) Solution (a)

In News - http://www.thehindu.com/sci-tech/energy-and-environment/New-Ginger-
species-with-medicinal-properties-found-in-Andamans/article17009348.ece

Q.5) Consider the following statements about Street Lighting National Programme (SLNP)

1. It is being implemented by Energy Efficiency Services Limited (EESL)


2. It is currently running in the South Delhi Municipal Corporation (SDMC) area with
special focus on the installation of LED light in parks, dark spots and high mast lights

Which of the following statements is/are correct?


IASbabas Daily Quiz
January 11, 2017

a) 1 and 2
b) 2 and 3
c) 1 and 3
d) All of the above

Q.5) Solution (c)

Street Lighting National Programme (SLNP) is currently running in the South Delhi Municipal
Corporation (SDMC) area

It is the Worlds Largest Street Light Replacement Programme

Implemented by the Energy Efficiency Services Limited (EESL), a joint venture under the
Ministry of Power, Government of India

In News - http://economictimes.indiatimes.com/industry/energy/power/worlds-largest-
led-street-light-replacement-project-launched/articleshow/56428162.cms
IASbabas Daily Quiz
January 12, 2017

Q.1) River Falgu flows in which of the following states?

a) Uttarakhand
b) Bihar
c) Himachal Pradesh
d) None of the above

Q.1) Solution (b)

The Phalgu or Falgu, a river that flows past Gaya, India in the Indian state of Bihar, is a
sacred river for Hindus and Buddhists. In ancient scriptures, it is called the Nairajan in
Sanskrit.

In News - http://www.thehindu.com/news/national/Six-rivers-flowing-above-danger-
mark-in-Bihar/article14569819.ece

Q.2) Orang National Park is located in

a) Assam
b) Manipur
c) Mizoram
d) Nagaland

Q.2) Solution (a)

It is located on the north bank of the Brahmaputra River in the Darrang and Sonitpur
districts of Assam, India, covers an area of 78.81 square kilometres (30.43 sq mi). It was
established as a sanctuary in 1985 and declared a national park on 13 April 1999. It is also
known as the mini Kaziranga National Park (IUCN site) since the two parks have a similar
landscape made up of marshes, streams and grasslands.

In News - http://timesofindia.indiatimes.com/home/environment/flora-fauna/Rhino-
dies-at-Rajiv-Gandhi-Orang-National-Park/articleshow/53738674.cms

Q.3) Consider the following statements about General Anti-Avoidance Rules (GAAR)

1. It contains provision allowing the government to prospectively tax overseas deals


involving local assets.
IASbabas Daily Quiz
January 12, 2017

2. It will come into effect from 1 April 2017 (assessment year 2018-19)

Which of the following statements is/are correct?

a) Only 1
b) Only 2
c) Both 1 and 2
d) Neither 1 nor 2

Q.3) Solution (c)

GAAR was part of the 2012-13 budget speech of the then finance minister Pranab
Mukherjee to check tax evasion and avoidance. However, its implementation was
repeatedly postponed because of the apprehensions expressed by foreign investors.

GAAR, which was originally to be implemented from 1 April 2014, will now come into effect
from 1 April 2017 (assessment year 2018-19). It contains provision allowing the government
to prospectively tax overseas deals involving local assets.

In News - http://www.livemint.com/Politics/gRrhVhtymINA4lMuDVwynI/Antiavoidance-
tax-rule-to-kick-in-from-April-2017.html

Q.4) Which of the following statements concerning Waqf is/are correct?

a) It is a permanent dedication of movable or immovable properties for religious, pious


or charitable purposes as recognized by Muslim Law, given by philanthropists
b) Central Waqf Council, India is an Indian statutory body established in 1964 by the
Government of India under Waqf Act, 1954 (now a sub section the Waqf Act, 1995)
c) A campaign has been launched by Ministry of Minority Affairs, National Waqf
Development Corporation (NAWADCO) and Central Waqf Council to utilise the Waqf
properties for socio-economic-educational empowerment of minority communities
specially Muslims
d) All of the above

Q.4) Solution (d)

A campaign has been launched by Ministry of Minority Affairs, National Waqf Development
Corporation (NAWADCO) and Central Waqf Council to utilise the Waqf properties for socio-
economic-educational empowerment of minority communities specially Muslims.
IASbabas Daily Quiz
January 12, 2017

The Minority Affairs Ministry has been working on different strategies for welfare of Muslim
community which include protection and development of Waqf properties.

Waqf is a permanent dedication of movable or immovable properties for religious, pious or


charitable purposes as recognized by Muslim Law, given by philanthropists.

The grant is known as mushrut-ul-khidmat, while a person making such dedication is known
as Waqf.

Central Waqf Council, India is an Indian statutory body established in 1964 by the
Government of India under Wakq Act, 1954 (now a sub section the Waqf Act, 1995)

In News - http://pib.nic.in/newsite/PrintRelease.aspx?relid=156196

Q.5) The Speaker can ask a member of the House to stop speaking and let another
member speak. This phenomenon is known as

a) decorum
b) crossing the floor
c) interpellation
d) yielding the floor

Q.5) Solution (d)

Yielding the floor - The speaker of the Lok sabha can ask a member of the house to stop
speaking and let another member speak.

Crossing the floor - Passing between the member addressing the House and the Chair which
is considered breach of Parliamentary etiquette.

Point of Order - A Member of Parliament may raise a point of order if he feels that the
proceedings of' the House do not follow the normal rules. The presiding officer decides
whether the point of order raised by the member should be allowed.

Calling Attention Motion - With the prior permission of the Speaker, any member of the
Parliament may call the attention of a Minister to a matter of urgent public importance. The
Minister may make a brief statement about the matter or he may ask for time to make a
statement later.
IASbabas Daily Quiz
January 13, 2017

Q.1) Recently, a place called Aswan was in news. It is located in which of the following
countries?

a) Morocco
b) Egypt
c) Algeria
d) Armenia

Q.1) Solution (b)

12 ancient Egyptian cemeteries discovered near Aswan by Swedish archaeologists

Human and animal remains were found in the cemeteries, which were discovered in the
Gabal al-Silsila or Chain of Mountains area 65 km (40.3 miles) north of Aswan.

In News- http://www.livemint.com/Science/g2llmmYaTQtsityQDhL8bO/12-ancient-Egyptian-
cemeteries-discovered-near-Aswan.html

Q.2) The Global Risks Report is published by

a) World Economic forum


b) World Bank
c) United Nations Security Council
d) None of the above

Q.2) Solution (a)

The Global Risks Report is an annual study published by the World Economic Forum ahead
of the Forums Annual Meeting in Davos, Switzerland. Based on the work of the Global Risk
Network, the report describes changes occurring in the global risks landscape from year to
year and identifies the global risks that could play a critical role in the upcoming year. The
report also explores the interconnectedness of risks, and considers how the strategies for
the mitigation of global risks might be structured.

Sources for the report include an assessment by several major insurance and reinsurance
companies and focus workshops, interviews and a survey of internationally recognised
experts. The report is intended to raise awareness about the need for a multi-stakeholder
approach to the mitigation of global risk.
IASbabas Daily Quiz
January 13, 2017

In News - http://www.livemint.com/Politics/FnIBE7U1ktYZtHhLeCizxO/World-Economic-
Forum-says-capitalism-needs-urgent-change.html

Q.3) Consider the following statements about Giant Metrewave Radio Telescope (GMRT)

1. It is an array of thirty fully steerable parabolic radio telescopes of 45 metre diameter,


observing at metre wavelengths, located near Pune
2. It is operated by the National Centre for Radio Astrophysics, a part of the Tata
Institute of Fundamental Research, Mumbai

Which of the following statements is/are correct?

a) Only 1
b) Only 2
c) Both 1 and 2
d) Neither 1 nor 2

Q.3) Solution (c)

The Giant Metrewave Radio Telescope (GMRT), located near Pune in India, is an array of
thirty fully steerable parabolic radio telescopes of 45 metre diameter, observing at metre
wavelengths. It is operated by the National Centre for Radio Astrophysics, a part of the Tata
Institute of Fundamental Research, Mumbai. At the time it was built, it was the world's
largest interferometric array offering a baseline of up to 25 kilometres (16 mi).

In News - http://indianexpress.com/article/technology/science/astronomers-discover-powerful-
cosmic-double-whammy-with-help-of-india-s-gmrt-4463561/

Q.4) Consider the following statements about India INX

1. The India INX is located in the country's first International Financial Services Centre
(IFSC), which is located in Mumbai
2. India INX will be the fastest international exchange in the world in terms of order
response time, with a median trade speed of four microseconds
3. The exchange can trade securities and products other than Indian rupees

Which of the following statements is/are correct?

a) 1 and 2
b) 2 and 3
IASbabas Daily Quiz
January 13, 2017

c) 1 and 3
d) All of the above

Q.4) Solution (b)

The India INX is located in the country's first International Financial Services Centre
(IFSC), which is located in GIFT City.
India INX will be the fastest international exchange in the world in terms of order
response time, with a median trade speed of four microseconds. In comparison, the
second fastest international exchange at Singapore has an order response time of 60
microseconds.
India INX at will be open for trading for 22 hours every day.
The exchange can trade securities and products other than Indian rupees. The
securities and products that could be traded on the India INX are: equity shares of
companies incorporated outside India, depository receipts, debt securities, currency
and interest rate derivatives, index based derivatives, commodity derivatives and
such other securities that may be allowed.

In News - http://www.business-standard.com/article/markets/bse-s-india-inx-10-things-to-know-
about-our-1st-international-exchange-117011000301_1.html

Q.5) Shekatkar panel is concerned with which of the following ministries?

a) Ministry of Finance
b) Ministry of Parliamentary Affairs
c) Ministry of Housing and Urban Poverty Alleviation
d) Mistry of Defence

Q.5) Solution (d)

Defence Ministry-appointed expert committee headed by Lt General D B Shekatkar (retd)


has recommended that the new post be of four-star rank, equivalent in rank to the current
chiefs of the Army, Navy and Air Force.

The Committee was established with a mandate for Enhancing Combat Capability and
Rebalancing Defence Expenditure.

In News - http://indianexpress.com/article/india/defence-reforms-shekatkar-panel-
recommends-four-star-rank-for-top-military-adviser-4468665/
IASbabas Daily Quiz
January 16, 2017

Q.1) World Employment and Social Outlook report is released by

a) World Bank
b) International Labour Organisation
c) World Economic Forum
d) United Nations Development Programme

Q.1) Solution (b)

World Employment and Social Outlook: Trends 2017 -


http://www.ilo.org/global/research/global-reports/weso/2017/WCMS_541211/lang--
en/index.htm

In News - http://www.livemint.com/Politics/qvL3mTzzJYkuqb5DPN0TwI/Unemployment-
in-India-to-increase-marginally-in-201718-UN.html

Q.2) Consider the following statements about European Southern Observatory (ESO)

1. The ESO Headquarters is located in Garching, near Munich, Germany


2. United Kingdom is not a member state of ESO

Which of the following statements is/are correct?

a) Only 1
b) Only 2
c) Both 1 and 2
d) Neither 1 nor 2

Q.2) Solution (a)

The ESO Headquarters are located in Garching, near Munich, Germany.

Member States - http://www.eso.org/public/about-eso/memberstates/

In News - http://indianexpress.com/article/technology/science/giant-telescope-in-chile-
to-seek-habitable-planets-in-alpha-centauri-4469652/
IASbabas Daily Quiz
January 16, 2017

Q.3) Which of the following statements is/are correct?

1. Alpha Centauri is the closest star system to the Solar System


2. It consists of three stars: the pair Alpha Centauri A and Alpha Centauri B together
with a small and faint red dwarf named Proxima Centauri

Select the correct code

a) Only 1
b) Only 2
c) Both 1 and 2
d) Neither 1 nor 2

Q.3) Solution (c)

It is the closest star system to the Solar System at a distance of 4.37 light-years

Centauri B together with a small and faint red dwarf, Alpha Centauri C (also named Proxima
Centauri.

In News - http://indianexpress.com/article/technology/science/giant-telescope-in-chile-
to-seek-habitable-planets-in-alpha-centauri-4469652/

Q.4) Consider the following statements about Asian Waterbird Census (AWC)

1. The AWC is part of the International Waterbird Census (IWC) of Wetlands


International
2. It is carried out simultaneously in 27 countries once a year

Which of the following statements is/are correct?

a) Only 1
b) Only 2
c) Both 1 and 2
d) Neither 1 nor 2

Q.4) Solution (c)

The AWC is part of the International Waterbird Census (IWC) of Wetlands International. It is
carried out simultaneously in 27 countries once a year. The survey is conducted across Asia
IASbabas Daily Quiz
January 16, 2017

in January by a national network of volunteers and conservation organisations in


coordination with national and State coordinators.

In News - http://www.thehindu.com/todays-paper/tp-national/tp-newdelhi/Fewer-
migratory-birds-at-Najafgarh-Lake-this-year/article17038763.ece

Q.5) Which of the following cities will host worlds first Gender Literature Festival?

a) Mumbai
b) Bangalore
c) Kolkata
d) None of the above

Q.5) Solution (d)

The worlds first Gender Literature Festival will be held in Patna in the second week of
April.

The Gender Resource Centre of Bihars Women Development Corporation will be organising
the festival.

In News - http://www.thehindu.com/todays-paper/tp-national/World%E2%80%99s-first-
gender-literature-fest-in-Patna/article17025447.ece
IASbabas Daily Quiz
January 17, 2017

Q.1) Who introduced the electron pair bond?

a) Ernest Rutherford
b) Gilbert N. Lewis
c) Niels Bohr
d) None of the above

Q.1) Solution (b)

G. N. Lewis introduced the electron pair bond.

He proposed that two atoms may share from one to six electrons forming single, double or
triple bonds.

He introduced the cubical atom and six postulates to understand their chemical behaviour.

In News - http://www.thehindu.com/todays-paper/tp-features/tp-educationplus/One-
hundred-years-of-the-%E2%80%98bond%E2%80%99/article17043374.ece

Q.2) Consider the following statements about Electronic Nicotine Delivery Systems (ENDS)

1. It is also called as e-cigarettes


2. It contains tobacco
3. It does not fall within the ambit of the Cigarettes and Other Tobacco Products
(Prohibition of Advertisement and Regulation of Trade and Commerce, Production,
Supply and Distribution) Act, 2003 (COTPA)

Which of the following statements is/are correct?

a) 1 and 3
b) 1 and 2
c) 2 and 3
d) All of the above

Q.2) Solution (a)

It is also called e-cigarettes, personal vaporizers, vape pens, ecigars, e-hookah, or vaping
devices, are products that produce an aerosolized mixture containing flavored liquids and
nicotine that is inhaled by the user.
IASbabas Daily Quiz
January 17, 2017

As e-cigarettes contain nicotine and not tobacco, they do not fall within the ambit of the
Cigarettes and Other Tobacco Products (Prohibition of Advertisement and Regulation of
Trade and Commerce, Production, Supply and Distribution) Act, 2003 (COTPA)

In News - http://www.thehindu.com/opinion/op-ed/Smoking-e-cigarettes-is-injurious-to-
health/article17041292.ece

Q.3) Development Administration Partnership (DPA) is under

a) Ministry of External Affairs


b) Ministry of Finance
c) Ministry of Corporate Affairs
d) Prime Ministers Office (PMO)

Q.3) Solution (a)

Development Administration Partnership (DPA) was created in the Ministry of External


Affairs in January 2012 to effectively handle Indias aid projects through the stages of
concept, launch, execution and completion.

DPA has three Divisions. Currently, DPA I deals with project appraisal and lines of credit;
DPA II deals with capacity building schemes, disaster relief, Indian Technical and Economic
Cooperation Programme and DPA III deals with project implementation.

Read More - https://www.mea.gov.in/development-partnership-administration.htm

In News - http://indianexpress.com/article/india/india-us-sign-mou-for-collaboration-in-
third-world-countries-4473555/

Q.4) Consider the following statements about Chagres river

1. It is a part of Panama Canal system


2. It is the only river in the world that empties into two oceans

Which of the following statement is/are correct?

a) Only 1
b) Only 2
c) Both 1 and 2
d) Neither 1 nor 2
IASbabas Daily Quiz
January 17, 2017

Q.4) Solution (c)

The Chagres River in central Panama, is the largest river in the Panama Canal's watershed.
The river is dammed twice, and the resulting reservoirsGatun Lake and Lake Alajuela
form an integral part of the canal and its water system. Although the river's natural course
runs northwest into the Caribbean Sea, its waters also flow, via the canal's locks, into the
Gulf of Panama to the south, entitling the Chagres to the unusual claim of drainage into two
oceans.

In News - http://www.livemint.com/Opinion/IGL9uBxrXiyt6lb3URfNkN/Water-as-a-force-
for-peace.html

Q.5) Which of the following statements about the International Committee of the Red
Cross (ICRC) is/are correct?

a) It is headquartered in Geneva
b) It is a three-time Nobel Prize Laureate
c) It is the only institution explicitly named under International Humanitarian Law (IHL)
as a controlling authority
d) All of the above

Q.5) Solution (d)

The International Committee of the Red Cross (ICRC) is a humanitarian institution based in
Geneva, Switzerland and a three-time Nobel Prize Laureate. State parties (signatories) to the
four Geneva Conventions of 1949 and their Additional Protocols of 1977 (Protocol I,
Protocol II) and 2005 have given the ICRC a mandate to protect victims of international and
internal armed conflicts. Such victims include war wounded, prisoners, refugees, civilians,
and other non-combatants.

The ICRC is part of the International Red Cross and Red Crescent Movement along with the
International Federation of Red Cross and Red Crescent Societies (IFRC) and 190 National
Societies. It is the oldest and most honoured organization within the Movement and one of
the most widely recognized organizations in the world, having won three Nobel Peace Prizes
in 1917, 1944, and 1963.

ICRC is the only institution explicitly named under International Humanitarian Law (IHL) as a
controlling authority. The legal mandate of the ICRC stems from the four Geneva
Conventions of 1949, as well as its own Statutes. The ICRC also undertakes tasks that are not
IASbabas Daily Quiz
January 17, 2017

specifically mandated by law, such as visiting political prisoners outside of conflict and
providing relief in natural disasters.

In News - http://www.livemint.com/Opinion/IGL9uBxrXiyt6lb3URfNkN/Water-as-a-force-
for-peace.html
IASbabas Daily Quiz
January 17, 2017
IASbabas Daily Quiz
January 18, 2017

Q.1) Recently, a mobile app 'SEZ India' which provides detailed information related to
special economic zones was launched. Consider the following statements

1. The app was launched under the Commerce ministrys e-Governance initiatives
2. It has four sections SEZ Information, SEZ Online Transaction, Trade Information and
Contact details

Which of the following statements is/are correct?

a) Only 1
b) Only 2
c) Both 1 and 2
d) Neither 1 nor 2

Q.1) Solution (c)

The Commerce Ministry has launched a mobile app SEZ India which provides detailed
information related to special economic zones.

The app was launched under the ministrys e-Governance initiatives.

It has four sections SEZ Information, SEZ Online Transaction, Trade Information and
Contact details

The SEZ online transaction is a dynamic submenu that tracks the Bill of Entry/Shipping Bill
processing status and also does verification.

The app also helps the Importers/Exporters to track the status of Bill of Entry/Shipping Bill
integration and processing in the EDI system of the ICEGATE

In News - http://www.financialexpress.com/industry/technology/commerce-ministry-
launches-sez-india-app-for-sezs/509072/

Q.2) Recently Falcon 9 rocket blasted off from California and placed a constellation of
satellites in orbit. It is developed by

a) Space X
b) Mishaal Aerospace
c) Virgin Galactic
d) PLD Space
IASbabas Daily Quiz
January 18, 2017

Q.2) Solution (a)

A SpaceX Falcon 9 rocket blasted off from California and placed a constellation of satellites
in orbit

A network of 10 satellites was launched as part of McLean, Virginia-based Iridium's project


to replace its existing network of satellites that provide global voice and data
communications.

Iridium plans six more Falcon 9 launches, each carrying 10 satellites, as part of a technology
upgrade

In News - http://indianexpress.com/article/technology/science/spacex-launches-first-
rocket-since-explosion-in-florida-4475067/

Q.3) Consider the following statements about Transports Internationaux Routiers (TIR)

1. It is the only global customs transit system that provides easy and smooth
movement of goods across borders in sealed compartments or containers under
customs control from the customs office of departure to the customs office of
destination
2. All members of INSTC are signatories to TIR Convention 1975
3. The conventions were adopted under the auspices of the United Nations Economic
Commission for Europe (UNECE)

Which of the following statements is/are correct?

a) 1 and 2
b) 1 and 3
c) 2 and 3
d) All of the above

Q.3) Solution (b)

India is gearing up to sign the Transports Internationaux Routiers (TIR), or the customs
convention on the international transport of goods.

TIR is the only global customs transit system that provides easy and smooth movement of
goods across borders in sealed compartments or containers under customs control from the
customs office of departure to the customs office of destination.
IASbabas Daily Quiz
January 18, 2017

It plays an important role in boosting regional connectivity and facilitating cross-border


trade flows

Since all members of INSTC, except India and Oman, are already signatories to TIR
Convention 1975, custom issues and common documentation issues could be quickly
resolved if India signs the convention and aligns its system with it.

The TIR system operates with certain parameters secure vehicles or container,
international guarantee chain, TIR carnet, reciprocal recognition of customs controls,
controlled access and TIR IT risk management tools.

These elements guarantee that goods travel across borders with minimum interference en
route and at the same time provide maximum safeguards to customs administration.

The TIR system has a globally accepted electronic control system for integrated transit
operations.

The conventions were adopted under the auspices of the United Nations Economic
Commission for Europe (UNECE)

In News - http://economictimes.indiatimes.com/news/economy/foreign-trade/india-
gears-up-to-ink-pact-for-global-customs-transit-system/articleshow/56578293.cms

Q.4) Which of the following statements about National Backward Classes Finance &
Development Corporation (NBCFDC) is/are correct?

a) It is under the aegis of Ministry of Social Justice and Empowerment


b) It provides financial assistance through State Channelizing Agencies (SCAs)
nominated by the State Governments/ UTs
c) The Loan & Employee Information Automation Project (LEAP) system would be used
by the National Backward Classes Finance & Development Corporation (NBCFDC) to
disburse loans electronically
d) All of the above

Q.4) Solution (d)

An online system for disbursement of loans to the people belonging to backward classes
was launched.
IASbabas Daily Quiz
January 18, 2017

The Loan & Employee Information Automation Project (LEAP) system would be used by the
National Backward Classes Finance & Development Corporation (NBCFDC) to disburse loans
electronically.

It is a Govt. of India Undertaking under the aegis of Ministry of Social Justice and
Empowerment

It was incorporated under Section 25 of the Companies Act 1956 on as a Company not for
profit with an objective to promote economic and developmental activities for the benefit
of Backward Classes and to assist the poorer section of these classes in skill development
and self-employment ventures.

It provides financial assistance through State Channelizing Agencies (SCAs) nominated by


the State Governments/ UTs. NBCFDC also provides Micro Financing through SCAs/ Self Help
Groups (SHGs).

In News - http://indianexpress.com/article/india/gehlot-launches-online-system-for-
disbursement-of-nbcfdc-loans-4472663/

Q.5) Consider the following statements about National Alliance against online Child sexual
abuse and exploitation

1. It was set up by the Ministry of Women & Child Development


2. It aims to develop a comprehensive outreach system to engage parents, schools,
communities, NGO partners and local governments (PRIs and ULBs) as well as police
and lawyers to ensure better implementation of the legal framework, policies,
national strategies and standards in relation to child protection and child rights
3. It will set up a multi-member secretariat based in MWCD with a portal inclusive of a
hotline for reporting and strengthening existing service delivery systems

Which of the following statements is/are correct?

a) 1 and 2
b) 1 and 3
c) 1, 2 and 3
d) Only 2

Q.5) Solution (c)

It was set up by the Ministry of Women & Child Development


IASbabas Daily Quiz
January 18, 2017

It aims to develop a comprehensive outreach system to engage parents, schools,


communities, NGO partners and local governments (PRIs and ULBs) as well as police and
lawyers to ensure better implementation of the legal framework, policies, national
strategies and standards in relation to child protection and child rights

To ensure protection of children from online sexual abuse, all stakeholders must work
together. In this context, the National Alliance on Child Sexual Abuse and Exploitation has
the following broad objectives:

Bring a common definition of child pornography including amendment of acts


(Information technology Act, POCSO Act).
Set up a multi-member secretariat based in MWCD with a portal inclusive of a
hotline for reporting and strengthening existing service delivery systems.
Provide a platform for Government/ NGOs and other child rights activists for
networking and information sharing.
Document and showcases success stories and best practices in terms of prevention
of online abuse and exploitation of children.
Inform and educate member organisations, parents, teachers, front line service
providers and children on the rights of the children and various issues related to
online child abuse and exploitation.
Provide a forum for advocacy for child rights and policy inputs based on research
and studies

In News - http://pib.nic.in/newsite/PrintRelease.aspx?relid=157384
IASbabas Daily Quiz
January 18, 2017

Q.1) Consider the following statements about asteroids

1. Psyche 16 is the largest S-type asteroid


2. S-type asteroids are the most common variety, forming around 75% of known
asteroids

Which of the following statements is/are correct?

a) Only 1
b) Only 2
c) Both 1 and 2
d) Neither 1 nor 2

Q.2) Solution (d)

16 Psyche is one of the ten most-massive asteroids in the asteroid belt. It is over 124
miles (200 km) in diameter and contains a little less than 1% of the mass of the entire
asteroid belt. Some astronomers believe it may be the exposed iron core of a
protoplanet. It is the most massive metallic M-type asteroid. Psyche was discovered
by the Italian astronomer Annibale de Gasparis on 17 March 1852 from Naples and
named after the Greek mythological figure Psyche.
16 Psyche is the largest M-type asteroid, and does appear to be metallic. 21 Lutetia,
an anomalous, probably non-metallic body, was the first M-type asteroid to be
imaged by a spacecraft when the Rosetta space probe visited it on July 10, 2010.
Another M-type, 216 Kleopatra, was imaged by radar by the Arecibo Observatory in
Puerto Rico and has a dog bone-like shape.
C-type asteroids are carbonaceous asteroids. They are the most common variety,
forming around 75% of known asteroids
S-type asteroids, or silicaceous asteroids, are of a stony composition, hence the
name. Approximately 17% of asteroids are of this type, making it the second most
common after the C-type.

In News - http://www.ibtimes.co.in/nasa-explore-giant-asteroid-16-psyche-solve-its-
mysteries-712004

Q.2) Raisina Dialogue is organised by

a) Ministry of External Affairs


b) Observer Research Foundation
c) Both (a) and (b)
IASbabas Daily Quiz
January 18, 2017

d) None of the above

Q.2) Solution (c)

The conference, organized like the Shangri-La Dialogue in Singapore, is held jointly by
Ministry of External Affairs, India and the Observer Research Foundation (ORF), an
independent think tank based in India.

In News - http://www.financialexpress.com/india-news/shashi-tharoor-hails-narendra-
modis-raisina-dialogue-welcomes-pms-foreign-policy/512251/

Q.3) Consider the following statements about Hybrid Cloud

1. It is a cloud computing environment which uses a mix of on-premises, private cloud


and third-party, public cloud services with orchestration between the two platforms.
2. It gives businesses greater flexibility and more data deployment options.
3. It can be used for big data processing

Which of the following statements is/are correct?

a) 1 and 2
b) 2 and 3
c) 1 and 3
d) All of the above

Q.3) Solution (d)

It is a cloud computing environment which uses a mix of on-premises, private cloud


and third-party, public cloud services with orchestration between the two platforms.
By allowing workloads to move between private and public clouds as computing
needs and costs change, hybrid cloud gives businesses greater flexibility and more
data deployment options.
Hybrid cloud is particularly valuable for dynamic or highly changeable workloads. It
can be used for big data processing. A company, for example, could use hybrid cloud
storage to retain its accumulated business, sales, test and other data, and then run
analytical queries in the public cloud, which can scale to support demanding
distributed computing tasks.

Read More - http://searchcloudcomputing.techtarget.com/definition/hybrid-cloud


IASbabas Daily Quiz
January 18, 2017

In News - http://indianexpress.com/article/technology/tech-news-technology/microsoft-
offers-to-collaborate-with-andhra-pradesh-on-e-governance-cyber-security-4479995/

Q.4) Mission 41K and Mission Raftaar is concerned with

a) Ministry of Road Transport and Highways


b) Ministry of Power
c) Ministry of Railways
d) Both (a) and (c)

Q.4) Solution (c)

Mission 41K - A plan that would save Rs 41,000 crore over 10 years through an integrated
energy management system

In this initiative, railways will electrify 24,000 km of rail tracks over the next five years by
doubling the annual rate of electrification from 2,000 km to 4,000 km in the next two years.

Mission Raftaar - Raise average speed of both passenger carrying trains and freight carrying
trains

In News - http://www.business-standard.com/article/economy-policy/suresh-prabhu-
unveils-mission-41k-energy-plan-for-railways-117011800230_1.html

Q.5) Einsteins Enigma or Black Holes in My Bubble Bath is written by

a) C.V. Vishveshwara
b) S. Ramanujan
c) C. V. Raman
d) S. N. Bose

Q.5) Solution (a)

C. V. Vishveshwara was an Indian scientist and black hole physicist. Specializing in Einstein's
General Relativity, he worked extensively on the theory of black holes and made major
contributions to this field of research since its very beginning.
IASbabas Daily Quiz
January 18, 2017

He has written several books to popularise his area of work that are widely read, one of
which is Einsteins Enigma, or, Black Holes in My Bubble Bath.

In News www.thehindu.com/sci-tech/science/C.V.-Vishveshwara-the-black-hole-man-
of-India-is-no-more/article17049343.ece
IASbabas Daily Quiz
January 18, 2017
IASbabas Daily Quiz
January 20, 2017

Q.1) The Taxation Laws (Second Amendment) Act, 2016 came into force on 15th on
December. Consider the following statements

1. It amends the Income Tax Act, 1961 and Finance Act, 2016
2. Pradhan Mantri Garib Kalyan Yojana (PMGKY) is a part of this amendment
3. Declarant of undisclosed income needs to pay 30% tax, 10% penalty and 33%
Pradhan Mantri Garib Kalyan Cess on the tax

Which of the following statements is/are correct?

a) 1 and 2
b) 2 and 3
c) 1 and 3
d) All of the above

Q.1) Solution (d)

The Pradhan Mantri Garib Kalyan Yojana (PMGKY) notified along with other provisions of
Taxation Laws (Second Amendment) Act, 2016 came into effect from 17 December 2016. It
will remain open until March 31, 2017.

Declaration under it can be made by any person in respect of undisclosed income in the
form of cash or deposits in an account with bank or post office or specified entity. Declarant
of undisclosed income needs to pay 30% tax, 10% penalty and 33% Pradhan Mantri Garib
Kalyan Cess on the tax, all of which add up to around 50%.

Read More - http://pib.nic.in/newsite/PrintRelease.aspx?relid=155589

In News - http://www.thehindu.com/todays-paper/tp-business/PMGKY-not-to-apply-for-
non-cash-income-Govt/article17057514.ece

Q.2) Consider the following statements about The Credit Guarantee Fund Scheme for
Micro and Small Enterprises (CGS)

1. It was launched to make available collateral-free credit to the micro and small
enterprise sector
2. Only the new enterprises are eligible to be covered under the scheme
3. The Ministry of Micro, Small and Medium Enterprises, and Small Industries
Development Bank of India (SIDBI), established a Trust named Credit Guarantee
Fund Trust for Micro and Small Enterprises (CGTMSE) to implement the Credit
Guarantee Fund Scheme for Micro and Small Enterprises
IASbabas Daily Quiz
January 20, 2017

Which of the following statements is/are correct?

a) 1 and 2
b) 1 and 3
c) 2 and 3
d) All of the above

Q.2) Solution (b)

The Credit Guarantee Fund Scheme for Micro and Small Enterprises (CGS) was launched by
the Government of India (GoI) to make available collateral-free credit to the micro and small
enterprise sector. Both the existing and the new enterprises are eligible to be covered under
the scheme. The Ministry of Micro, Small and Medium Enterprises, GoI and Small Industries
Development Bank of India (SIDBI), established a Trust named Credit Guarantee Fund Trust
for Micro and Small Enterprises (CGTMSE) to implement the Credit Guarantee Fund Scheme
for Micro and Small Enterprises.

In News - http://www.thehindu.com/todays-paper/tp-business/Cabinet-boosts-corpus-
for-small-enterprises/article17057511.ece

Q.3) Which of the following countries is/are members of East African Community (EAC)?

1. Burundi
2. Rwanda
3. Senegal
4. Angola
5. Uganda

Select the correct code:

a) 1, 2 and 5
b) 1, 3, 4 and 5
c) 1, 3 and 5
d) 1, 2 and 4

Q.3) Solution (a)


IASbabas Daily Quiz
January 20, 2017

The East African Community (EAC) is the regional intergovernmental organisation of the
Republics of Kenya, Uganda, the United Republic of Tanzania, Republic of Burundi and
Republic of Rwanda with its headquarters in Arusha, Tanzania.

The EAC aims at widening and deepening co-operation among the partner states and other
regional economic communities in, among others, political, economic and social fields for
their mutual benefit.

The East African Community (EAC) is the regional intergovernmental organization of the
Republics of Burundi, Kenya, Rwanda, Uganda and the United Republic of Tanzania with its
Headquarters in Arusha, Tanzania. The Treaty for Establishment of the East African
Community was signed on 30th November 1999 and entered into force on 7th July 2000
following its ratification by the Original 3 Partner States Kenya, Uganda and Tanzania. The
Republic of Rwanda and the Republic of Burundi acceded to the EAC Treaty on 18th June
2007 and became full Members of the Community with effect from 1st July 2007.

In News - http://www.thehindu.com/todays-paper/tp-opinion/Reaching-out-to-
Africa/article17020813.ece

Q.4) Agenda 2063 is concerned with

a) Sustainable Development Goals


b) African Union
c) World Fare Trade Organization
d) None of the above

Q.4) Solution (b)

Agenda 2063 is both a Vision and an Action Plan. It is a call for action to all segments of
African society to work together to build a prosperous and united Africa based on shared
values and a common destiny.

In News - http://www.thehindu.com/todays-paper/tp-opinion/Reaching-out-to-
Africa/article17020813.ece

Q.5) Recently President Pranab Mukherjee unveiled the Wanderer Car used by
_________________ during his Great Escape from Kolkata to the Gomoh railway station
between January 16 and 18, 1941. Select the correct code.
IASbabas Daily Quiz
January 20, 2017

a) Vikram Sarabhai
b) J.R.D Tata
c) Ghanshyam Das Birla
d) None of the above

Q.5) Solution (d)

President Pranab Mukherjee unveiled the Wanderer Car used by Netaji Subhas Chandra
Bose during his Great Escape from Kolkata to the Gomoh railway station between January
16 and 18, 1941. The car was restored by Audi.

In News - http://www.thehindu.com/news/national/Pranab-unveils-Netaji%E2%80%99s-
restored-car/article17057272.ece
IASbabas Daily Quiz
January 23, 2017

Q.1) Consider the following statements about Conference of Central and State Statistical
Organizations (COCSSO)

1. The 24th COCSSO was organised by Ministry of Statistics and Programme


Implementation (MoSPI) in collaboration with the Directorate of Economics and
Statistics (DES), Government of Maharashtra
2. The theme of this years Conference is Agriculture and Farmers Welfare

Which of the following statements is/are correct?

a) Only 1
b) Only 2
c) Both 1 and 2
d) Neither 1 nor 2

Q.1) Solution (c)

As per the recommendations of the National Statistical Commission, Ministry of Statistics &
Programme Implementation organizes a COCSSO every year, which provides a platform for
the Central and State Statisticians to exchange views and discuss common issues relating to
statistical activities.

The 24th COCSSO was organised in Nagpur by Ministry of Statistics and Programme
Implementation (MoSPI) in collaboration with the Directorate of Economics and Statistics
(DES), Government of Maharashtra

The theme of this years Conference is Agriculture and Farmers Welfare

In News - http://pib.nic.in/newsite/PrintRelease.aspx?relid=157481

Q.2) Recently Jones Lang LaSalles released its fourth annual City Momentum Index of
cities around the world. Consider the following statements

1. The Index tracks the speed of change of a city's economy and commercial real estate
market
2. Bengaluru has emerged as the most dynamic city
3. 5 of the top 10 cities are from Asia

Which of the following statements is/are correct?

a) 1 and 2
IASbabas Daily Quiz
January 23, 2017

b) 2 and 3
c) 1 and 3
d) All of the above

Q.2) Solution (d)

The Index tracks the speed of change of a city's economy and commercial real estate
market. It covers 134 major established and emerging business hubs and identifies cities
that have the potential to maintain the greatest dynamism over the short and long term.

Bengaluru has emerged as the most dynamic city in Jones Lang LaSalles fourth annual City
Momentum Index of cities around the world. Bengaluru is followed by Ho Chi Minh City of
Vietnam and Silicon Valley in the U.S. Other cities in Top 10 category include Shanghai,
Hyderabad, London, Austin, Hanoi, Boston and Nairobi.

JLL City Momentum Index - http://www.jll.com/cities-research/city-momentum

In News - http://www.thehindubusinessline.com/news/six-indian-cities-in-worlds-most-
dynamic-list/article9487780.ece

Q.3) Which of the following statements about International Convention on Standards of


Training, Certification and Watch keeping for Seafarers (STWC) is/are correct?

a) STCW was adopted in 1978 by conference at the International Maritime


Organization (IMO) in London, and entered into force in 1984
b) The 1978 STCW Convention was the first to establish basic requirements on training,
certification and watch keeping for seafarers on an international level
c) It applies to ships of non-party States when visiting ports of States which are Parties
to the Convention
d) All of the above

Q.3) Solution (d)

The International Convention on Standards of Training, Certification and Watchkeeping for


Seafarers (or STCW), 1978 sets qualification standards for masters, officers and watch
personnel on seagoing merchant ships.
IASbabas Daily Quiz
January 23, 2017

STCW was adopted in 1978 by conference at the International Maritime Organization (IMO)
in London, and entered into force in 1984. The Convention was significantly amended in
1995.

The 1978 STCW Convention was the first to establish basic requirements on training,
certification and watchkeeping for seafarers on an international level. Previously the
standards of training, certification and watchkeeping of officers and ratings were
established by individual governments, usually without reference to practices in other
countries. As a result, standards and procedures varied widely, even though shipping is
extremely international by nature.

The Convention prescribes minimum standards relating to training, certification and


watchkeeping for seafarers which countries are obliged to meet or exceed.

One important feature of the Convention is that it applies to ships of non-party States when
visiting ports of States which are Parties to the Convention.

In News - http://pib.nic.in/newsite/PrintRelease.aspx?relid=157441

Q.4) The Chagos Archipelago is located in

a) Atlantic Ocean
b) Indian Ocean
c) Pacific Ocean
d) Arctic Ocean

Q.4) Solution (b)

The British Foreign Secretary has sought Indian assistance in resolving current tensions
between the U.K., the U.S. and Mauritius over the future of the U.S. military base Diego
Garcia, and the Indian Ocean Chagos Archipelago, amid a warning from Mauritius last year
that it would push to take the matter to the International Court of Justice.

The Chagos Islands referred to by the British as the British Indian Ocean Territory, but
which is not recognised as such by Mauritius is home to the U.S. military base Diego
Garcia. In the 1960s and 1970s, inhabitants were removed from the islands. Tensions
remain, with Mauritius maintaining that the archipelago remains its integral part.

In News - http://www.thehindu.com/news/international/U.K.-seeks-Indian-help-in-
resolving-Chagos-Archipelago-dispute/article17063926.ece
IASbabas Daily Quiz
January 23, 2017

Q.5) Consider the following statements about Lean Manufacturing Competitiveness


Scheme

1. It is a component of National Manufacturing Competitiveness Programme (NMCP)


2. It is basically an initiative to reduce "waste" in manufacturing
3. National Productivity Council (NPC) was selected as National Monitoring and
Implementing Unit (NMIU) for facilitating implementation and monitoring of the
Scheme

Which of the following statements is/are correct?

a) 1 and 2
b) 2 and 3
c) 1, 2 and 3
d) 1 and 3

Q.5) Solution (c)

The objectives of the Scheme are to enhance the manufacturing competitiveness of MSMEs
through the application of various Lean Manufacturing (LM) techniques by;

Reducing waste;
Increasing productivity;
Introducing innovative practices for improving overall competitiveness;
Inculcating good management systems; and
Imbibing a culture of continuous improvement.

The Lean Manufacturing Competitiveness Scheme was started as a pilot phase in 2009 for
100 Mini Clusters (10 or so manufacturing MSME units) in 11th Five Year Plan. National
Productivity Council (NPC) was selected as National Monitoring and Implementing Unit
(NMIU) for facilitating implementation and monitoring of the Scheme.

The Scheme was up-scaled in September, 2013 considering the recommendations of the
evaluation report conducted by Quality Council of India (QCI).

In News - http://www.business-standard.com/article/government-press-
release/interaction-with-beneficiaries-of-up-scaled-revised-2013-lean-manufacturing-
competitiveness-117012001291_1.html
IASbabas Daily Quiz
January 24, 2017

Q.1) Which of the following statements about the Vaquita is/are correct?

1. It is a rare species of porpoise


2. It is the most endangered cetacean in the world
3. It endemic to the northern part of the Gulf of California

Select the correct code:

a) 1 and 2
b) 2 and 3
c) 1 and 3
d) All of the above

Q.1) Solution (d)

An ambitious, emergency plan to help save the vaquita, a rare species of porpoise, from
extinction in the northern Gulf of California has been recommended by the International
Committee for the Recovery of the Vaquita (CIRVA). The plan involves relocating some of
the remaining vaquitas to a temporary sanctuary, while crucial efforts aimed at eliminating
illegal fishing and removing gill nets from their environment continue. The emergency action
plan will be led by the Mexican government and supported by a consortium of marine
mammal experts from more than a dozen organisations around the world.

Since the baiji (Lipotes vexillifer) is believed to have gone extinct by 2006, the vaquita has
taken on the title of the most endangered cetacean in the world. It is listed as critically
endangered because the estimated number of individuals dropped below 100 in 2014,
putting it in imminent danger of extinction. That number was updated to approximately 60
in May 2016, leading to the conclusion that the species is headed for extinction within five
years unless further conservation efforts are undertaken.

In News - http://www.thehindu.com/sci-tech/health/article17075761.ece

Q.2) Consider the following statements about Wolf 1061c

1. It is an exoplanet orbiting within the habitable zone of the red dwarf star Wolf 1061
2. It is the second-closest known potentially habitable exoplanet to Earth, after
Proxima Centauri b
3. It has an atmosphere more similar to Venus

Which of the following statements is/are correct?


IASbabas Daily Quiz
January 24, 2017

a) 1 and 2
b) 2 and 3
c) 1, 2 and 3
d) Only 1

Q.2) Solution (c)

Wolf 1061c or WL 1061c is an exoplanet orbiting within the habitable zone of the red dwarf
star Wolf 1061 in the constellation Ophiuchus, about 13.8 light years from Earth, making it
the second-closest known potentially habitable exoplanet to Earth, after Proxima Centauri
b. It has an atmosphere more similar to Venus.

But the climate of Wolf 1061c is quite chaotic compared to earth, since the orbit around its
star changes at a much faster rate than earth. The earth also experiences climate change
due to change in its orbit around the sun which resulted in ice age previously.

It is the second planet in order from its host star in a triple planetary system, and has an
orbital period of 17.9 days. Wolf 1061c is classified as a super-Earth exoplanet as its
estimated radius is greater than 1.5 R.

In News - http://indianexpress.com/article/technology/science/wolf-1061c-exoplanet-
could-be-the-next-habitable-planet-after-earth-4484780/

Q.3) Which of the following is/are correctly matched?

1. Arunachal Pradesh :: Yak Dance


2. Manipur :: Hojagiri
3. Tripura :: Lai Haroba
4. Odisha :: Dola Jatra

Select the correct code:

a) 1 and 4
b) 1, 3 and 4
c) 2 and 3
d) Only 1

Q.3) Solution (a)


IASbabas Daily Quiz
January 24, 2017

Arunachal Pradesh (Yak Dance) - It is one of the most famous pantomimes of the Mahayana
sect of Buddhist Tribes of Arunachal Pradesh

Manipur (Lai Haroba) It is one of the oldest ritualistic theatres of the world preserved by
the Meitei community of Manipur. It is celebrated to worship local deities to bring
prosperity and well-being. It is translated as Happiness of the Gods.

Tripura (Hojagiri) - It is a popular dance form of Reang Tribe. It is performed during bihu,
the most popular festival of reang tribes

Odisha (DolaJatra) - A popular festival celebrated in the state which signifies the journey of
Goddess Radha& Lord Krishna for ultimate union in the tradition of Bhakti cult.

In News - http://www.hindustantimes.com/india-news/skill-india-beti-bachao-beti-
padhao-themes-for-68th-republic-day-tableaux/story-TyK1HK2ThOBX4uGKeVKWeM.html

Q.4) Which of the following statements is/are correct?

a) Anadromous fishes hatch or are born in marine habitats, but migrate to freshwater
areas where they spend the majority of their lives growing and maturing.
b) Catadromous fishes are the opposite of catadromous fishes in that hatching and a
juvenile period occur in freshwater.
c) Both (a) and (b)
d) Neither (a) nor (b)

Q.4) Solution (d)

Diadromous fishes describe species that spend part of their lives in freshwater and part in
saltwater. There are two categories of diadromous fishes, catadromous and anadromous .

Catadromous fishes hatch or are born in marine habitats, but migrate to freshwater areas
where they spend the majority of their lives growing and maturing.

Anadromous fishes are the opposite of catadromous fishes in that hatching and a juvenile
period occur in freshwater.

Q.5) The 713th death anniversary of Sufi saint Nizamuddin Auliya was recently observed
by reciting qawwals in the Nizamuddin dargah. Consider the following statements

1. Nizamuddin Auliya is a sufi saint belonging to the Qadiriyya order


IASbabas Daily Quiz
January 24, 2017

2. His notable disciple was Amir Khusrow

Which of the following statements is/are correct?

a) Only 1
b) Only 2
c) Both 1 and 2
d) Neither 1 nor 2

Q.5) Solution (b)

Nizamuddin Auliya is a sufi saint belonging to the Chisti order.

Nizamuddin Auliyas predecessors were Qutbuddin Bakhthiyar Kaki, Khwaja Moinuddin


Chisti. His notable disciple was Amir Khusrow.

The Chisti order is a Sunni Sufi order within the Sufi tradition of Islam. The order believed in
drawing close to God through renunciation of the world and service to humanity.

This order is primarily followed in Afghanistan and Indian Subcontinent.

The Chisti order is the first of the four main Sufi orders Chisti, Qadiriyya, Suhrawardiyya
and Naqshbandi order.

In News - http://www.hindustantimes.com/art-and-culture/meet-the-nizami-bandhu-
qawwali-s-rockstars/story-NlHuHmoLvv2omhiPRbOqHP.html
IASbabas Daily Quiz
January 26, 2017

Q.1) Consider the following statements about Mission Indradhanush

1. It was launched by the Ministry of Health and Family Welfare


2. It aims to cover all those children by 2020 who are either unvaccinated, or are
partially vaccinated against seven vaccine preventable diseases which include
diphtheria, whooping cough, tetanus, polio, tuberculosis, measles and hepatitis B

Which of the following statements is/are correct?

a) Only 1
b) Only 2
c) Both 1 and 2
d) Neither 1 nor 2

Q.1) Solution (c)

Mission Indradhanush was launched by the Ministry of Health and Family Welfare,
Government of India on December 25, 2014. Between 2009-2013 immunization coverage
has increased from 61% to 65%, indicating only 1% increase in coverage every year. To
accelerate the process of immunization by covering 5% and more children every year,
Indradhanush mission has been adopted to achieve target of full coverage by 2020.

The Mission Indradhanush, depicting seven colours of the rainbow, aims to cover all those
children by 2020 who are either unvaccinated, or are partially vaccinated against seven
vaccine preventable diseases which include diphtheria, whooping cough, tetanus, polio,
tuberculosis, measles and hepatitis B.

In News - http://indianexpress.com/article/opinion/columns/ministry-of-health-child-
immunisation-mission-indradhanush-4491673/

Q.2) Consider the following statements about Predatory Pricing

1. It is the act of setting prices low in an attempt to eliminate the competition


2. It makes markets more vulnerable to a monopoly

Which of the following statements is/are incorrect?

a) Only 1
b) Only 2
c) Both 1 and 2
d) Neither 1 nor 2
IASbabas Daily Quiz
January 26, 2017

Q.2) Solution (d)

Predatory pricing is the act of setting prices low in an attempt to eliminate the competition.
Predatory pricing is illegal under anti-trust laws, as it makes markets more vulnerable to a
monopoly. Companies may engage in a variety of activities that intend to drive out
competitors, such as create barriers to entry for new competitors or unethical production
methods to minimize costs.

Read More - http://www.thehindubusinessline.com/opinion/predatory-pricing-and-the-


law/article6541819.ece

In News - http://www.livemint.com/Industry/sywa17TWNWqHJ6rPh16OZP/Predatory-
pricing-of-new-entrants-like-Reliance-Jio-is-hur.html

Q.3) Which of the following statements is/are correct?

1. National Voters Day is observed on 25th of January


2. A non-resident Indian can vote in his/her hometown after registering as an Overseas
Voter
3. When an Indian becomes a citizen of another country, he/she automatically loses
the right to vote
4. Overseas Citizens of India, who are citizens of another country, do not have the right
to vote

Select the correct code

a) 1, 2 and 4
b) 1, 2 3 and 4
c) 2, 3 and 4
d) 2 and 3

Q.3) Solution (b)

National Voters Day is observed on 25th of January. On this day in 1950, the Election
Commission of India was founded. The ECI observes National Voters Day to create
awareness on voting and voter rights.

A non-resident Indian can vote in his/her hometown after registering as an Overseas Voter

When an Indian becomes a citizen of another country, he/she automatically loses the right
to vote
IASbabas Daily Quiz
January 26, 2017

Overseas Citizens of India, who are citizens of another country, do not have the right to vote

In News - http://www.thehindu.com/news/national/National-Voter%E2%80%99s-Day-
What-are-your-voting-rights/article17092722.ece

Q.4) Consider the following statements about Varishtha Pension Bima Yojana 2017

1. The scheme is meant to provide an assured pension based on a guaranteed rate of


return of 10.25% per annum for ten years, with an option to opt for pension on a
monthly/quarterly/half-yearly or annual basis
2. It will be implemented by Life Insurance Corporation of India (LIC)

Which of the following statements is/are correct?

a) Only 1
b) Only 2
c) Both 1 and 2
d) Neither 1 nor 2

Q.4) Solution (b)

The scheme will be implemented through Life Insurance Corporation of India (LIC) during
the current financial year to provide social security during old age and protect elderly
persons aged 60 years and above against a future fall in their interest income due to
uncertain market conditions.

The scheme is meant to provide an assured pension based on a guaranteed rate of return of
8% per annum for ten years, with an option to opt for pension on a monthly/quarterly/half-
yearly or annual basis. The difference between the return generated by LIC and the assured
return of 8% per annum would be borne by Government of India as subsidy on an annual
basis. VPBY-2017 is proposed to be open for subscription for a period of one year from the
date of launch.

In News - http://www.thehindu.com/business/Economy/Govt-launches-Varishtha-
Pension-Bima-Yojana/article17089241.ece

Q.5) Disguised Unemployment is generally found in

a) Agriculture
b) Trade
IASbabas Daily Quiz
January 26, 2017

c) Industry
d) Transport

Q.5) Solution (a)

Disguised unemployment exists where part of the labor force is either left without work or
is working in a redundant manner where worker productivity is essentially zero. It is
unemployment that does not affect aggregate output. An economy demonstrates disguised
unemployment when productivity is low and too many workers are filling too few jobs.

Source: http://www.tutionteacher.in/what-do-you-understand-by-disguised-
unemployment-explain-with-an-example-each-from-the-urban-and-rural-areas/

In News - http://www.financialexpress.com/budget/budget-2017-big-bets-arun-jaitley-
must-make-it-attractive-for-private-sector-to-invest-in-infrastructure-housing/522657/
IASbabas Daily Quiz
January 27, 2017

Q.1) The Ministry of Home Affairs will issue the industrial licences for defence
manufacturing which includes electronic aerospace and defence equipment. Consider the
following statements

1. Previously it was issued by Department of Industrial Policy and Promotion (DIPP)


under Ministry of Commerce and Industry
2. With the notification of Arms Rule, 2016, Items configured for military use will be
handled by Home Ministry instead of DIPP

Which of the following statements is/are correct?

a) Only 1
b) Only 2
c) Both 1 and 2
d) Neither 1 nor 2

Q.1) Solution (c)

The Ministry of Home Affairs will issue the industrial licences for defence manufacturing
which includes electronic aerospace and defence equipment. Previously it was issued by
Department of Industrial Policy and Promotion (DIPP) under Ministry of Commerce and
Industry. The participation of private sector in defence manufacturing was allowed since
2001 subject to licensing from DIPP under Industries (Development and Regulation) Act,
1951. However with the notification of Arms Rule, 2016, Items configured for military use
will be handled by Home Ministry instead of DIPP.

In News - http://www.thehindubusinessline.com/economy/policy/home-ministry-will-
now-issue-defence-industrial-licences/article9500149.ece

Q.2) Consider the following statements about National Clean Energy Fund (NCEF)

1. It is created for funding research and innovative projects in clean energy


technologies
2. An Inter-Ministerial Group (IMG) chaired by Finance Secretary approves the
projects/schemes eligible for financing under the NCEF
3. Creation of NCEF was announced in the Union Budget 2015-16

Which of the following statements is/are correct?

a) 1 and 2
b) 2 and 3
IASbabas Daily Quiz
January 27, 2017

c) 1 and 3
d) All of the above

Q.2) Solution (a)

The National Clean Energy Fund (NCEF) is a fund created in 2010-11 using the carbon tax -
clean energy cess - for funding research and innovative projects in clean energy
technologies of public sector or private sector entities, upto the extent of 40% of the total
project cost. Assistance is available as a loan or as a viability gap funding, as deemed fit by
the Inter-Ministerial group, which decides on the merits of such projects.

The Fund is designed as a non-lapsable fund under Public Accounts and with its secretariat
in Plan Finance II Division, Department of Expenditure, Ministry of Finance.

Creation of NCEF was announced in the Union Budget 2010-11.

An Inter-Ministerial Group, chaired by the Finance Secretary in Ministry of Finance (and


comprising of Secretaries of Departments of Expenditure and Revenue at Ministry of
Finance, Principal Scientific Advisor to the Government of India, a representative of Planning
Commission and a Representatives of Ministry sponsoring the proposal and other Ministries
concerned with that specific proposal) recommends projects eligible for funding under
NCEF.

In News - http://www.business-standard.com/article/economy-policy/most-of-the-rs-54-
000-cr-national-clean-energy-fund-remains-unused-117012500013_1.html

Q.3) Consider the following statements about Investor-state dispute settlement (ISDS)

1. It is a system through which individual companies can sue countries for alleged
discriminatory practices
2. ISDS is found in the Energy Charter Treaty of which India is a signatory
3. It is contained in NAFTA and proposed TPP

Which of the following statements is/are correct?

a) 1 and 2
b) 1 and 3
c) 2 and 3
d) All of the above
IASbabas Daily Quiz
January 27, 2017

Q.3) Solution (b)

ISDS is an instrument of international public law, it is a neutral international arbitration


procedure.

ISDS seeks to provide an impartial, law-based approach to resolve conflicts.

It is a system through which individual companies can sue countries for alleged
discriminatory practices.

It is contained in a number of bilateral investment treaties, in certain international trade


treaties, such as NAFTA (chapter 11), and the proposed TPP (chapters 9 and 28) and CETA
(sections 3 and 4) agreements.

ISDS is also found in international investment agreements, such as the Energy Charter
Treaty.

The Energy Charter Treaty (ECT) is an international agreement which establishes a


multilateral framework for cross-border cooperation in the energy industry. The treaty
covers all aspects of commercial energy activities including trade, transit, investments and
energy efficiency. The treaty is legally binding, including dispute resolution procedures. India
is not a member of Energy Charter Conference.

In News - http://www.thehindu.com/business/India-rejects-attempts-by-EU-Canada-for-
global-investment-agreement/article17083034.ece

Q.4) Canary Islands is located in

a) Pacific Ocean
b) Indian Ocean
c) Atlantic Ocean
d) None of the above

Q.4) Solution (c)

The Canary Islands also known as the Canaries, are an archipelago and autonomous
community of Spain located on the Atlantic Ocean, 100 kilometres (62 miles) west of
Morocco. The Canaries are among the outermost regions (OMR) of the European Union
proper. It is also one of the eight regions with special consideration of historical nationality
recognized as such by the Spanish Government.
IASbabas Daily Quiz
January 27, 2017

In News - http://www.thehindu.com/todays-paper/tp-national/Global-travel-sector-
zooms-in-on-lucrative-gay-market/article17080984.ece

Q.5) Which of the following statements about vote-on-account (VOA) is/are correct?

a) A VOA in the strict sense deals only with the expenditure side of the government's
budget, whereas an interim budget has to include both expenditure and receipts
b) A VOA cannot be for a period longer than six
c) No changes are made to tax and duty structures and no new schemes are announced
in vote-on-account
d) All of the above

Q.5) Solution (d)

What is Vote-on-account?

Vote-on-account literally means a vote on the accounts of the government. Usually,


the annual budget is presented by the end of February after which it is discussed
details of the budget are scrutinized by a Parliamentary committee and it is finally
passed by mid-May.
However, this time, this could be in the middle of elections or another government
could be in power depending on the election schedule.
During elections and till a new government takes over, the caretaker government
needs funds for various routine items of expenditure like staff salaries without
which there would be a financial crisis. According to the Constitution, the
government cannot spend any money without Parliament's approval.
Hence, vote-on-account is taken whereby a government gets parliamentary approval
to run the government for a few months, using funds drawn from the Consolidated
Fund of India.

How is a vote-on-account different from the full budget or an interim budget?

While the words vote-on-account and interim budget are often interchangeably
used, a vote-on-account in the strict sense deals only with the expenditure side of
the government's budget, whereas an interim budget has to include both
expenditure and receipts.
Generally, a vote-on-account is for two or three months, usually till the time it is
replaced by a regular budget.
IASbabas Daily Quiz
January 27, 2017

It cannot be for a period longer than six months as the Constitution stipulates that
the gap between two Parliament sittings cannot be more than six months.
A regular full budget is a complete statement on the financial position of the
government for a full year based on expenditures during the period and proposals
for financing them. Thus, it gives details of how money is to be spent and how it will
be raised by the government.

Why ever have a vote-on-account and not a full-fledged budget?

Constitutionally, there is no distinction between a caretaker government and a


regular one. The government could technically present a full budget.
However, by convention, a government that is at the end of its tenure opts for a
vote-on-account since it is regarded as improper that an outgoing government
should impose its policies on its successor.
There is also the fear that in election years a full budget would tempt governments
to resort to populism while ignoring financial prudence.
Interim budgets have also been used by governments taking office just before the
financial year begins to get Parliamentary approval for immediate spending, giving
them time to work out a more though-out budget later in the year.

By convention, what are the restrictions on a vote-on-account?

Vote on account gives the revised estimates of expenditure incurred by the


government and revised estimates of government revenue from different sources in
the financial year coming to an end.
These estimates provide an assessment of how efficiently the government spent its
resources and how effective its policies of mobilizing tax and non-tax revenues were.
Typically, no changes are made to tax and duty structures and no new schemes are
announced. However it can extend coverage or allocate more money to an existing
scheme. The finance minister can also use the vote-on-account speech to give
indications of what he would like to do if given an opportunity after the elections.
Thus, it is often used to indicate his intentions on economic policy just before
elections and he can make many promises, something most finance ministers have
done in their vote-on-account speeches.

In News - http://timesofindia.indiatimes.com/elections/news/dont-announce-any-
scheme-for-poll-bound-states-in-union-budget-ec-tells-
government/articleshow/56742740.cms
IASbabas Daily Quiz
January 30, 2017

Q.1) Which of the following statements is/are correct?

a) Ministry of Health and Family Welfare is the nodal ministry for handling epidemics,
decision making, advisory body and emergency medical relief provider
b) Ministry of Home Affairs (MHA) is the nodal ministry for Biological Warfare and
partners with Ministry of Health and Family Welfare in its management
c) National Institute of Communicable Diseases (NICD) is the nodal agency for
investigating outbreaks of biological disasters
d) All of the above

Q.1) Solution (d)

All the statements are correct.

Source: Yojana January 2017 (Disaster Management)

Topic - Biological Disasters: Causes and Way Forward

Q.2) Consider the following statements about Asian Ministerial Conference for Disaster
Risk Reduction (AMCDRR)

1. AMCDRR is a biennial conference jointly organized by different Asian countries and


the United Nations Office for Disaster Risk Reduction (UNISDR)
2. This first Asian Ministerial Conference for Disaster Risk Reduction after the advent of
the Sendai Framework will be hosted by India

Which of the following statements is/are correct?

a) Only 1
b) Only 2
c) Both 1 and 2
d) Neither 1 nor 2

Q.2) Solution (c)

In Asia, the regional platform mainly consists of the Asian Ministerial Conferences on
Disaster Risk Reduction (AMCDRR) and the ISDR Asia Partnership (IAP) forum as its
mechanism for consultation and technical support. Established in 2005, the AMCDRR is a
biennial conference jointly organized by different Asian countries and the United Nations
Office for Disaster Risk Reduction (UNISDR).
IASbabas Daily Quiz
January 30, 2017

The AMCDRR serves as a forum for stakeholders to take a shared responsibility and make
actionable commitments towards implementation of DRR in the Asia region through the
exchange of experiences on successful practices and innovative approaches in reducing and
managing disaster risk. So far, countries in Asia in collaboration with UNISDR, have
organized six AMCDRR conferences. The Previous hosts were the Peoples Republic of China
(Beijing, 2005); Republic of India (New Delhi, 2007); Malaysia (Kuala Lumpur, 2008);
Republic of Korea (Incheon, 2010); Indonesia (Yogyakarta, 2012); and Thailand (Bangkok,
2014).

This first Asian Ministerial Conference for Disaster Risk Reduction after the advent of the
Sendai Framework was hosted by the Government of India in November 2016. As a followup
from the 6th Asian Ministerial Conference outcome (2014) and as a requirement of the
Sendai Framework, the intended outcome of the conference in India will be to adopt an
Asian Regional Plan for Implementation of the Sendai Framework.

Source: Yojana January 2017 (Disaster Management)

Topic - Post Sendai Initiatives and Way Forward

Q.3) National Crisis Management Committee (NCMC) is chaired by

a) Cabinet Secretary
b) Home Secretary, Ministry of Home Affairs
c) Prime Minister
d) None of the above

Q.3) Solution (a)

Read More - http://www.ndmindia.nic.in/management/ncmc.html

Source: Yojana January 2017 (Disaster Management)

Topic Managing Disaster Risk

Q.4) Consider the following statements about United Nations Disaster Assessment and
Coordination (UNDAC)

1. It is part of the international emergency response system for sudden-onset


emergencies.
IASbabas Daily Quiz
January 30, 2017

2. The office for the Coordination of Humanitarian Affairs (OCHA) at the request of the
government affected by a disaster dispatches a UNDAC team to the country within
12 to 48 hours anywhere in the world

Which of the following statements is/are correct?

a) Only 1
b) Only 2
c) Both 1 and 2
d) Neither 1 nor 2

Q.4) Solution (c)

The United Nations Disaster Assessment and Coordination (UNDAC) is part of the
international emergency response system for sudden-onset emergencies.

UNDAC was created in 1993. It is designed to help the United Nations and governments of
disaster-affected countries during the first phase of a sudden-onset emergency. UNDAC also
assists in the coordination of incoming international relief at national level and/or at the site
of the emergency.

UNDAC teams can deploy at short notice (12-48 hours) anywhere in the world. They are
provided free of charge to the disaster-affected country, and deployed upon the request of
the United Nations Resident or Humanitarian Coordinator and/or the affected Government.

Read More - http://www.unocha.org/what-we-do/coordination-tools/undac/overview

Source: Yojana January 2017 (Disaster Management)

Q.5) Consider the following statements about National Cyclone Risk Mitigation Project
(NCRMP)

1. National Disaster Management Authority (NDMA) under the aegis of Ministry of


Home Affairs (MHA) will implement the Project in coordination with participating
State Governments and the National Institute for Disaster Management (NIDM)
2. NCRMP Phase 1 covers states of Goa,Gujarat,Karnataka,Kerela,Maharashtra and
West Bengal

Which of the following statements is/are correct?

a) Only 1
b) Only 2
IASbabas Daily Quiz
January 30, 2017

c) Both 1 and 2
d) Neither 1 nor 2

Q.5) Solution (a)

The Government of India has initiated the National Cyclone Risk Mitigation Project (NCRMP)
with a view to address cyclone risks in the country. The overall objective of the Project is to
undertake suitable structural and non-structural measures to mitigate the effects of
cyclones in the coastal states and UTs of India.

National Disaster Management Authority (NDMA) under the aegis of Ministry of Home
Affairs (MHA) will implement the Project in coordination with participating State
Governments and the National Institute for Disaster Management (NIDM).

The Project has identified 13 cyclones prone States and Union Territories (UTs), with varying
levels of vulnerability. These States/UT have further been classified into two categories,
based on the frequency of occurrence of cyclone, size of population and the existing
institutional mechanism for disaster management.

NCRMP Phase-II Government of India has approved Phase-II of NCRMP in July,2015 for five
years up to March, 2020 covering States of Goa, Gujarat, Karnataka, Kerala, Maharashtra
and West Bengal

Source: Yojana January 2017 (Disaster Management)

Topic Training and Capacity Building


IASbabas Daily Quiz
January 31, 2017

Q.1) Global Leprosy Strategy 20162020 is launched by

a) WHO
b) UNDP
c) UNICEF
d) None of the above

Q.1) Solution (a)

In 2016, WHO launched the Global Leprosy Strategy 20162020: Accelerating towards a
leprosy-free world, with the aim of reinvigorating efforts to control leprosy and avert
disabilities, especially among children still affected by the disease in endemic countries.

World Leprosy Day is observed on the last Sunday of January since 1954.

In News - http://www.thehindu.com/sci-tech/health/Why-India-needs-to-step-up-its-
fight/article17109549.ece

Q.2) Consider the following statements about Vanadium dioxide

1. It contradicts the principle of Wiedemann-Franz Law


2. It conducts electricity without conducting heat

Which of the following statements is/are correct?

a) Only 1
b) Only 2
c) Both 1 and 2
d) Neither 1 nor 2

Q.2) Solution (c)

The relationship between electrical and thermal conductivity is governed by the


Wiedemann-Franz Law, which states that good conductors of electricity are also good
conductors of heat.

That is not the case for metallic vanadium dioxide, a material already noted for its unusual
ability to switch from an insulator to a metal when it reaches 67 degrees Celsius.
IASbabas Daily Quiz
January 31, 2017

Electrons in vanadium dioxide can conduct electricity without conducting heat. The findings
could lead to a wide range of applications, such as thermoelectric systems that convert
waste heat from engines and appliances into electricity.

Q.3) Which of the following statements is/are correct?

1. Multilateral Competent Authority Agreement for Country-by- Country Reporting


(CbC MCAA) allows signatories to bilaterally and automatically exchange country-by-
country reports with each other, as contemplated by Base Erosion and Profit Shifting
(BEPS) Action Plan.
2. Recently Mauritius, Gabon, Hungary, Indonesia, Lithuania, Malta and the Russia have
signed the (CbC MCAA)
3. India is yet to sign CbC MCAA

Select the correct code:

a) 1 and 3
b) 1 and 2
c) 2 and 3
d) All of the above

Q.3) Solution (b)

CbC MCAA allows signatories to bilaterally and automatically exchange country-by-country


reports with each other, as contemplated by Base Erosion and Profit Shifting (BEPS) Action
Plan.

It will help ensure that tax administrations obtain a better understanding of how MNEs
structure their operations, while also ensuring that the confidentiality and appropriate use
of such information is safeguarded.

BEPS refers to tax avoidance strategies that exploit gaps and mismatches in tax rules to
artificially shift profits to low or no-tax locations.

Under the inclusive framework, over 100 countries and jurisdictions are collaborating to
implement the BEPS measures and tackle such instances

The Multilateral Competent Authority Agreement for automatic exchange of country-by-


country reports (CbC MCAA) was signed by India at Beijing on May 12 2016.
IASbabas Daily Quiz
January 31, 2017

Besides India, five other countries Canada, China, Iceland, Israel and New Zealand also
signed the pact on the same day.

In News - http://www.business-standard.com/article/pti-stories/mauritius-joins-global-
efforts-to-curb-profit-shifting-by-mnes-117012700798_1.html

Q.4) TROPEX is a defence exercise between

a) India and Nepal


b) Nepal and Bhutan
c) India and Sri Lanka
d) None of the above

Q.4) Solution (d)

Theatre Readiness Operational Exercise (TROPEX) is an annual inter-service exercise.

The month-long exercise will have ships and aircraft of both the Western and Eastern Naval
Commands, as also assets from the Indian Air Force, Indian Army and the Indian Coast
Guard exercising together.

In News - http://indianexpress.com/article/india/indias-inter-service-military-exercise-
tropex-kicks-off-4494692/

Q.5) Consider the following statements about Guyots

1. Guyot is an isolated underwater volcanic mountain with a flat top over 200 m below
the surface of the sea
2. They are most commonly found in the Arctic Ocean
3. They often have a high level of biological productivity because they provide habitats
for many species of plants and animals

Which of the following statements is/are correct?

a) 1 and 2
b) 1 and 3
c) Only 1
d) All of the above
IASbabas Daily Quiz
January 31, 2017

Q.5) Solution (b)

A guyot, also known as a tablemount or seamount, is an isolated underwater volcanic


mountain with a flat top over 200 m below the surface of the sea.

The diameters of these flat summits can exceed 10 km.

Guyots are most commonly found in the Pacific Ocean.

It is formed by gradual subsidence through stages from fringed reefed mountain, coral atoll,
and finally a flat topped submerged mountain.

Seamounts often have a high level of biological productivity because they provide habitats
for many species of plants and animals. Over 200 species of sea creatures have been
observed at a single guyot in the New England Seamount. Seamounts are great locations to
discover new species because each seamount houses different types of animals, including
many that can only be found in guyot habitats.

Das könnte Ihnen auch gefallen